SlideShare a Scribd company logo
1 of 587
CARDIOVASCULAR
ATHEROSCLEROSIS
Atherosclerosis is the leading cause of 
Morbidity and Mortality in Western Society 
Most Common Factors: 
Hypertension 
Smoking 
Hypercholesterolemia 
Diabetes
Risk Factors 
• Dyslipidemia: When in excess, LDL accumulates in subendothelial space and can start to 
damage the intima, initiating and perpetuating development of atherosclerotic lesions. 
Treatments slow progression of atherosclerotic plaques. 
• Smoking: Tobacco smoking could lead to atherosclerotic disease in several ways, including 
enhanced oxidative modification of LDL, decreased circulating HDL levels, endothelial 
dysfunction owing to tissue hypoxia and increased oxidant stress, increased platelet 
adhesiveness among others. 
• Hypertension: Hypertension injures vascular endothelium and may increase the 
permeability of the vessel wall to lipoproteins. Antihypertensive results like lipid-reducing 
treatments, halting of atherosclerotic progression 
• DM: High blood sugar may result in nonenzymatic glycosylation of lipoproteins (which 
enhances uptake of cholesterol by scavenger macrophages, as described earlier), or to a 
prothrombotic tendency and antifibrinolytic state that is often present. Diabetics frequently 
have impaired endothelial function, gauged by the reduced bioavailability of NO and 
increased leukocyte adhesion. Managing DM may halt progression 
• Elevated CRP: Atherosclerosis is thought to arise from an inflammatory reaction; CRP is a 
marker of inflammation.
ATHEROSCLEROSIS
Atherosclerosis is an INTIMAL PROCESS
Progression of Atherosclerosis
What are the steps of Atherosclerosis? 
• Endothelial Activation 
• LDL, monocytes, endothelium 
• Foam Cells 
• Oxidized LDL, macrophages 
• Fibrous Cap 
• SMCs 
• Calcification 
• Ulceration 
• Thrombosis
Name the important molecules involved in 
mediating atherosclerosis 
• 1. Monocyte attaches to VCAM-1 
• 2. MCP-1 is released to attract more monocytes 
• 3. Macrophages release MPO to oxidize LDL 
• 4. Scavenger receptor on macrophage allows LDL uptake 
• 5. Lipid core releases PDGF to stimulate SMCs 
• 6. SMCs release MMP, plaque is destabilized
What are the important complications of 
atherosclerosis? 
• Occlusion of vessel 
• Thrombus 
• Ulceration and hemorrhage 
• Atheroembolism 
• Narrowing of lumen (stenosis) 
• Weakening of wall (aneurysm)
Atherosclerosis end results
Name the Complications
Dangers of Complicated Plaques 
• Simple atheromas have a fibrous cap that conveys a great deal of 
plaque stability. Thus with simple atheromas, patients may present 
with arterial stenosis resulting in hypoperfusion but are unlikely to 
present with thrombotic or embolic events. 
• Complicated atheromas can have an ulcerated cap, which expose 
procoagulants in plaque to circulation increasing chance of thrombus 
at the site or lead to greater occlusion of vessel. Rupture of a plaque 
in complicated atheromas can also lead to hemorrhage. Also 
complicated atheromas can have calcifications, which can increase 
the fragility of the plaque and thus likelihood of thrombotic events.
Atheroembolism will show cholesterol 
plaques
Hyperlipidemia Drugs
Mechanism Competitively inhibit HMG-CoA reductase: 
(1) Decreases intracellular cholesterol  induces SREBP  
increases expression of LDL-R 
(2) VLDL and IDL are cleared more rapidly due to cross-recognition 
with hepatic LDL-R 
(3) Hepatic VLDL production falls due to reduced cholesterol 
availability  reduced LDL and triglycerides 
Modify platelets and endothelium (e.g., enhanced NO 
synthesis) 
Suppress inflammation 
Effects Decreases LDL 18-55% 
Decreases TG 7-30% 
Increases HDL 5-15% (unclear mechanism) 
Side Effects Myopathy (increased w/ niacin, fibrates), hepatotoxicity, drug 
interactions (CYP3A4 inhibition: macrolides, azoles, HIV 
protease inhibitors) 
Statins
Mechanism Inhibits NPC1L1 at brush border of epithelial cells in 
small intestine  reduced chylomicron production  
less cholesterol delivered to liver  compensatory 
increase in hepatic LDL-R  increased LDL clearance 
Effects Decreases LDL 18% 
Has additive effect w/ statins and fibrates on LDL 
Side Effects Muscle weakness (slightly higher w/ statin), 
transaminitis (slightly higher w/ statin) 
Ezetimibe
Bile Acid Resins 
Mechanism (+)-charged amines bind (-)-charged bile acids and prevent 
recycling in liver 
hepatic cholesterol (FXR, CYP7A ) LDL-R 
UNLIKE statins, new cholesterol production is stimulated (b/c 
HMG-CoA reductase is not inhibited)  VLDL production  
serum TGs 
Effects Decreases LDL 15-30% 
Increases HDL 4% 
No change/slight increase in TGs 
Side Effects GI, drug interactions (fat-soluble drugs: esp warfarin, digoxin), 
absorption of fat-soluble vitamins, pancreatitis, cholesterol 
gallstones 
*Not absorbed systemically, so no systemic side effects*
Niacin (Vitamin B3) 
Mechanism Decreases lipolysis in adipose tissue  less FAs available for 
TG synthesis in liver 
Decreases VLDL synthesis, so less LDL 
Increases HDL by decreasing hepatic removal of HDL 
Effects Decreases LDL 5-25% 
Increases HDL 15-30% 
Decreases TGs 20-50% 
Side Effects Cutaneous flushing (due to prostaglandins; take aspirin), GI 
(nausea, PUD), hepatotoxicity, insulin resistance and 
hyperglycemia (caution w/ diabetics), gout (raises serum uric 
acid levels), myopathy (increases w/ statin)
Mechanism Activate PPARα-RXR  
(1) Enhanced oxidation of FAs in liver and muscle  
decreased TG levels  decreased VLDL 
(2) Increased expression of LPL 
(3) Increased rate of HDL-mediated reverse cholesterol 
transport (due to apo AI transcription) 
Effects Decreases LDL 5-20% 
Increases HDL 10-20% 
Decreases TGs 20-50% 
*Larger decreases in TGs and increases in HDL than statins. 
Side Effects GI (dyspepsia, abdominal pain, diarrhea), cholesterol 
gallstones, myopathy (increased w/ liver and kidney 
dysfunction; worse w/ statins), augment effects of oral 
hypoglycemic drugs (avoid in diabetes) 
Fibrates 
Gemfibrozil inhibits glucuronidation of most statins, which can increase statin-related 
side effects. Fenofibrate does not.
Gemofibrizil vs. Fenofibrate 
• Drugs that block CYP3A4 slow statin metabolism, increasing the risk 
of side effects. 
• Gemfibrozil inhibits the glucuronidation of statins and increases the 
chances of side effects. 
• Choose Fenofibrate in patients taking Statins.
Fish Oil (omega-3 FA) 
Mechanism Not well defined, ?agonist of PPARα, 
?binds enzymes in TG synthesis but 
cannot be utilized 
Effects Variable effect on LDL 
Increases HDL 9% 
Decreases TGs 50% 
Side 
Effects 
Minimal, may prolong bleeding time 
(caution use w/ NSAIDs, ASA, 
warfarin), caution w/ shellfish 
hypersensitivity
Major SE of Hyperlipidemia Tx
4 Major Statin Treatment Groups 
• Individuals with clinical atherosclerotic cardiovascular disease (ASCVD). 
• Tx recommendations: 
• High-intensity statin if < 75 years old 
• Moderate-intensity statin if > 75 years old (or < 75 y/o and not a candidate for high-intensity) 
• Individuals with primary elevations of LDL-C >190 mg/dL. 
• Tx recommendation: High-intensity statin (or moderate if not a candidate) 
• Individuals w/ diabetes aged 40-75 yrs with LDL-C 70-189 mg/dL and w/o 
clinical ASCVD. 
• Tx recommendation: 
• Estimated 10-year risk of ASCVD < 7.5%: Moderate-intensity statin 
• Estimated 10-year risk of ASCVD > 7.5%: High intensity statin 
• Individuals w/o clinical ASCVD or diabetes with LDL-C 70-189 mg/dL and 
10 year ASCVD risk > 7.5%. 
• Tx recommendation: Moderate or high intensity statin
HDL… Good or Bad? 
• HDL is an important player in Reverse Cholesterol Transport and 
removes cholesterol from circulation. Also there is an inverse 
correlation between HDL levels and CVD risk- high HDL = low CVD 
risk 
• Controversy: Two clinical trials aimed to increase HDL failed in phase 
III due to adverse off target effects and lack of efficacy. Another 
clinical trial using niacin that increases HDL showed no added benefit.
Lifestyle Guidelines 
• Consume a dietary pattern that emphasizes intake of vegetables, fruits, 
and whole grains; includes low-fat dairy products, poultry, fish, legumes, 
nontropical vegetable oils, and nuts; and limits intake of sweets, sugar-sweetened 
beverages, and red meats. 
• Adapt this dietary pattern to appropriate calorie requirements, personal 
and cultural food preferences, and nutrition therapy for other medical 
conditions (including diabetes). 
• Achieve this pattern by following plans such as the DASH dietary pattern, 
the US Department of Agriculture (USDA) Food Pattern, or the AHA Diet. 
• Aim for a dietary pattern that achieves 5% to 6% of calories from 
saturated fat. 
• Reduce percentage of calories from saturated fat. 
• Reduce percentage of calories from trans fat. 
• Engage in aerobic physical activity to reduce LDL-C and non–HDL-C: 3 
to 4 sessions per week, lasting on average 40 minutes per session, and 
involving moderate- to vigorous- intensity physical activity. 
•
PCSK9 
• PCSK9 is a protein that promotes the degradation of LDL receptors 
and decreases the amount of LDL receptors on cell surface. Any 
manipulation that will decrease the functionality or availability of 
PCSK9 will result in increased LDL receptors on cell surface and thus 
lower circulating LDL levels. 
• Potential Future RNA interference drug
CARDIOVASCULAR 
HEMODYNAMICS
Define mean arterial pressure (MAP) and 
describe how changes in cardiac output 
(CO), systemic vascular resistance (SVR), 
and central venous pressure (CVP) affect 
MAP. 
• Mean arterial pressure (MAP) - the average arterial pressure during a 
single cardiac cycle. 
• MAP=(CO x SVR) + CVP 
• MAP = PDIAS + PPULSE/3 
• MAP is not the arithmetic mean of PDIAS and PSYS since the heart 
spends twice as long in diastole than systole under resting conditions 
• Increases in the cardiac output (CO), systemic vascular resistance 
(SVR), and central venous pressure (SVR) will increase the mean 
arterial pressure.
Compare the pressures, flows, and 
resistances in the pulmonary circulation 
with those in the systemic circulation. 
• Pressure: Pulmonary < Systemic 
• Flow: Pulmonary = Systemic 
• Resistance: Pulmonary < Systemic 
• Pulmonary v. Systemic- Pressures in the pulmonary circulation are 
not as high as the pressures found in the systemic circulation. The 
pressure in the pulmonary circulation is 25/10mmHg. The blood 
volume output has to be equal on both sides of the heart so as to 
ensure that there are no major blood volume changes between the 
pulmonary and systemic circulations. Although is there is resistance 
within the pulmonary circulation, it is less than the resistance found in 
system circulation.
Define central venous pressure (CVP) 
and how this pressure relates to stroke 
volume and cardiac output (CO). 
• Central venous pressure (CVP) is the pressure found in the vena 
cava near the right atrium. 
• Same in same out. 
• CVP is a measure of how much blood is going into the heart, which is 
equal to how much blood is coming out of the heart. Blood flows from 
peripheral venous pool (PVP) to central venous pool (CVP). The 
difference in pressure between PVP and CVP is the driving force 
pushing blood into the heart. By increasing the difference in pressure 
between the two systems, you create a larger flow from PVP to CVP, 
which results in more blood going into the right atrium  GREATER 
STROKE VOLUME AND CARDIAC OUTPUT
Explain how each of the following affects 
CVP: Blood volume, venous compliance, 
gravity, respiration, and muscle 
contraction. 
• Blood volume: increased blood volume increased venous return  increased CO 
• Venous compliance: decrease in venous compliance increases central venous 
pressure. 
• Gravity: Blood pools in LE  momentary decrease in venous return/CO before 
compensatory mechanisms kick in 
• Respiration: during inspiration, the intrapleural pressure decreases. This leads to a 
decrease in CVP, increase in pressure gradient, increase in venous return/CO 
• Muscle Contraction: muscle contraction in the leg helps facilitate the movement of 
blood from the veins in the lower extremities. This mechanism works against gravity to 
help increase CVP and increase CO.
Define the formula that relates blood flow 
through an arteriole to (a) pressure 
difference down the length of an arteriole 
and (b) resistance to that flow. 
• The formula that relates blood flow (F) through an arteriole to 
pressure (P) down a length and resistance (R) is 
• Q = ΔP/R
Explain what key factors are responsible 
for the pressure difference and resistance 
to flow. 
• Resistance depends upon viscosity of blood (n), vessel length (l) and 
vessel radius (r) 
• R = (n*l)/ (r^4) 
• Q = Delta P * (r ^4)/ (n*l) 
• Flow is directly related to pressure difference and inversely related to 
resistance
Explain why the decrease in pressure 
across arterioles is much greater than the 
pressure drop across other vessel types. 
• Resistance is related to radius to the 4th power, vessel length and 
viscosity of the blood. 
• Resistance is strongly related to the diameter of the vessel lumen: the 
smaller the diameter of the vessel lumen, the greater the resistance. 
Taken as an individual vessel, the individual capillary would exhibit 
greater resistance than the individual arteriole, because the diameter of 
the typical capillary lumen is smaller, but, if one looks at the entire 
vascular system as a whole, much more of the resistance of the system 
is contributed by arterioles than by capillaries 
• There are so many more capillaries and with many capillaries in parallel, 
that allows collectively a lot of blood flow whereas arterioles tend to be 
found individually and they also tend to be much longer than capillaries 
(which also increases resistance). Thus, In the larger system, arterioles 
are the greater contributor to resistance.
Define the formula that relates resistance 
to blood flow to (a) length of the arteriole, 
(b) viscosity of blood, and (c) internal 
radius of the arteriole. 
• Resistance = (viscosity * length)/ (radius^4) 
• The radius has the greatest effect (note it is to the fourth power), so 
vasoconstriction and vasodilation greatly affect resistance.
Explain which one of these factors has the 
greatest influence on the resistance to 
blood flow and what in turn are the 
greatest influences on it. 
• Radius has greatest effect on flow because it is to the fourth power 
• Blood volume and vessel compliance, vasodilation/constriction, and 
pathologic sclerosis may affect radius.
Predict the relative changes in flow 
through an arteriole caused by changes in 
arteriole length, arteriolar radius, fluid 
viscosity, and pressure difference. 
• Using the equations Q = ΔP/R and Resistance = (viscosity * 
length)/radius4 
• Increase in length → more resistance → less flow 
• Increase in radius → less resistance → more flow 
• Increase in viscosity → more resistance → less flow 
• Increase in pressure difference → more flow
Differentiate between the pressures and 
forces that influence the caliber of intra-alveolar 
capillaries and those that 
influence extra-alveolar resistance 
vessels. 
• Intra-alveolar capillaries are very closely associated with alveoli, which expand 
upon inhalation. This causes the capillaries to also stretch, thus decreasing their 
diameter. This results in a greater resistance. 
• For capillaries, increased lung volume means greater resistance 
• Extra-alveolar resistance vessels, on the other hand, are tethered to the pleural 
cavity. When the lung volume expands, the connective tissue attaching them to 
the pleural cavity pulls on their walls to expand their diameter 
• For bigger vessels, decreased lung volume means greater resistance 
• There is a sweet spot in the middle where the lung volume causes the resistance 
to be the least for both capillaries and the resistance vessels and this is the 
“working lung volume.”
Explain the effects of gravity on 
pulmonary blood flow. 
• When standing up, CVP drops since venous blood pools in the legs. 
This decreases venous return and cardiac output, also decreasing 
pulmonary blood flow. Also when the person is standing up, the blood 
flow is lowest at the apex of the lung and highest at the base.
Describe how pulmonary blood flow 
varies from the base of the lungs to the 
apex. 
• Due to effects of gravity, the base of the lung receives more blood 
than the apex. This leads to poor “V/Q” matching, since ventilation (V) 
is not matched to blood flow (Q) at different areas of the lung. 
• The V/Q ratio at the top of the lungs is high, while the V/Q ratio at the 
base of the lungs is low.
Describe the changes in pulmonary 
vascular resistance when the pressures in 
pulmonary arteries and pulmonary veins 
increase. 
• Q = ΔP/R; 
• If pressure increases, resistance also increases proportionately in 
order to keep the flow constant. However, pulmonary circulation has a 
much lower resistance because the vessels have thinner walls, have 
less smooth muscle, are more distensible and they can do 
“recruitment”. Therefore, the pressure can remain low while 
maintaining the same flow
Describe the roles of recruitment and 
distension in decreasing pulmonary 
vascular resistance. 
• The CO of the right and left sides of the heart are equal. Because 
the pressures on the systemic side is so much greater than on the 
pulmonary side, the pulmonary system must have much less 
resistance. Pulmonary circulation is able to accomplish this lower 
resistance with thinner vessel walls, less smooth muscle in the vessel 
walls, greater distensibility, and recruitment of non-perfused vessels. 
• Recruitment is adding new vessels in parallel circuit to lower the total 
resistance, while distension is decreasing the resistance in each 
vessel by expanding its diameter
Use the Fick Principle to estimate 
pulmonary blood flow. 
• Assumption: in the steady state, the cardiac output of the L and R 
ventricles are equal 
• O2 consumption = Blood flow ( Arterial-venous O2 Difference)
Explain the mechanism and significance 
of hypoxic vasoconstriction. 
• When PAO2 is normal (around 100 mmHg), O2 diffuses from the 
alveoli into the nearby arteriolar smooth muscle cells, keeping the 
arterioles relatively relaxed and dilated. When PAO2 decreases to 
below 70 mmHg, the smooth muscle cells can recognize the reduced 
amount of O2 coming in from the alveoli. This causes them to 
contract, reducing the amount of pulmonary blood flow to that region 
• Significance: 
• This reduces pulmonary blood flow to poorly ventilated areas. 
Pulmonary blood flow is directed away from poorly ventilated regions 
of the lung and redirected to other, better oxygenated regions
COAGULATION CASCADE
PT- Extrinsic Pathway 
PTT – Intrinsic Pathway (TENET)
Summarize the mechanism for the 
initiation of a blood clot and identify the 
essential enzymes involved in this 
mechanism. 
• Tissue Factor (TF) is usually in the subendothelial membrane on 
smooth muscle and collagen. When tissue is injured, it is released, 
activating the coagulation cascade. This activates the Extrinsic 
Pathway by forming a complex with factor 7a.
Predict the effect of vitamin K deficiency 
on the coagulation system 
• Factors II, VII, IX, and X require gamma carboxylation 
from Vitamin K in order to bind Ca on the surface of a clot. Vitamin K is 
absorbed in the intestine and metabolized in the liver. Therefore, people 
with end-stage liver disease or people who are not adequately absorbing 
Vit K in their intestines will see prolongation of the PT. We see a 
prolongation of the PT because the Extrinsic Pathway is most affected. 
• Vitamin K is also required for activation of anticoagulant proteins C and 
S. These proteins are the first to drop in Vit K deficiency, so we often see 
a transient pro-thrombotic state before the clotting factors drop. 
• (This is especially important to remember with administration of heparin).
Explain why a patient with end stage liver 
disease may have abnormal coagulation 
function. 
• Vitamin K is metabolized in the liver. Therefore, people with end 
stage liver disease are going to have a Vitamin K deficiency and won’t 
be able to activate Factors II, VII, IX, and X.
Describe the conversion of fibrinogen to 
fibrin and the role of factor XIII in this 
reaction. 
• Thrombin (IIa) converts fibrinogen (I) into fibrin (Ia), which is 
necessary to form a clot. Once fibrinogen gets converted into fibrin, it 
creates fibrin bonds and forms a fibrin mesh with crosslinks. That 
mesh gets activated and covalently bonds in the presence of factor 
XIII (activated by thrombin). Factor XIII stabilizes the clot in normal 
hemostasis. Factor XIII does not prolong PT or PTT.
Outline the steps of fibrinolysis and 
identify the inhibitors of fibrin degradation. 
• Tissue plasminogen activator (t-PA) is released from damaged 
vessels and cleaves plasminogen to the active enzyme plasmin. 
• In the circulation, plasminogen activator inhibitors 1 and 2 rapidly 
inactivate t-PA. 
• However, t-PA binds to fibrin locally at the site of release, and 
converts fibrin-bound plasminogen to plasmin. Plasmin splits both 
fibrinogen and fibrin into degradation products (D-dimer), and if this 
occurs at the site of a thrombus it produces lysis of the clot matrix.
Diagram the formation of the D-dimer and 
explain its utility in diagnosis of venous 
thromboembolic disease. 
• D-dimer: cross-linked regions of fibrin that have been degraded - “you 
have a clot, you’ve broken it down, and now you have d-dimer in your 
blood” - specific for fibrinolysis (however, not fibrinogenolysis) and 
useful for excluding thrombosis
Given values for various clotting factor 
concentrations, be able to predict which 
screening tests of coagulation will be 
abnormal. 
• PT (prothrombin time): tests function of common and extrinsic 
pathway (factors I (Fibrinogen), II (Prothrombin) , V, VII, and X). 
Defect in any of these factors => increased PT 
• PTT (partial thromboblastin time): tests function of common and 
intrinsic pathway (factors I, II, XII, XI, IX, VIII, X). Defect in any of 
these factors => increased PTT 
• Vitamin K deficiency => increase in both because factors II, VII, IX, X 
require Vitamin K, so all pathways are affected).
Explain how activated protein C and 
antithrombin act as inhibitors of 
coagulation. 
• Thrombomodulin (TM), which is normally present on the endothelial 
cells, binds thrombin. 
• Protein C (a vitamin K dependent factor that is normally found in the 
blood) binds to the Thrombin/TM complex, and becomes activated as 
Activated Protein C. 
• Protein S binds activated Protein C (APC), accelerating its activity. 
• APC inhibits coagulation by inactivating Factors V and VIII.
CV ELECTRICAL ACTIVITY
Define the roles of (a) If Na+-channels, (b) 
K+-channels, (c) voltage-gated Ca2+- 
channels, and (d) K-Ach channels, in 
pacemaker activity of SA cells. 
• a) If Na+-channels: Voltage and receptor gated. Contribute to phase 4 pacemaker funny 
current (repolarized state to depolarized state) due to a slow inward movement of Na+. 
• Note: “diastolic depolarization” refers to pacemaker phase/phase 4 and it represents the non-contracting 
time between heartbeats (diastole) 
• b) K+ channels: outflux of K+ in phase 3 contributes to repolarization 
• c) Voltage-gated Ca2+ channels: influx of Ca2+ during phase 0 contributes to depolarization 
• d) K-ACh channels: ACh activated K+ channels are activated by ACh and adenosine and are 
G-protein coupled. They slow SA nodal firing upon VAGAL stimulus. 
• 1. ACh is released from vagal nerve terminals (parasympathetic innervation of heart) near SA node 
• 2. ACh binds to ACh activated K+ channels and K+ channels open 
• 3. Resting potential approaches reversal potential for K+, about -90 mV 
• 4. As a result, SA cells take longer to reach threshold for firing
Describe the effects and mechanisms by 
which NE and Ach affect the activity of 
these ions channels and therefore HR. 
• NE: causes positive chronotropy (increase in heart rate). 
• Sympathetic activation of SA node increases slope of phase 4 and lowers 
threshold, increasing pacemaker frequency 
• NE released by sympathetic adrenergic nerves binds to 훃1 adrenoreceptors (훃 
1AR above) coupled to a stimulatory Gs-protein which activates adenylyl cyclase 
and increases cAMP 
• This leads to an increase in funny current (If) and an earlier opening of Ca2+ 
channels, both of which increase rate of depolarization 
• Ach: causes negative chronotropy (slowing of heart rate). 
• Vagal nerve stimulation releases ACh at SA node which decreases slope of phase 
4 by inhibiting funny currents, hyperpolarizing the cell, and increasing threshold 
voltage required to trigger phase 0. 
• ACh binds to M2 receptors and decreases cAMP via inhibitory Gi protein 
• ACh also activates K-ACh channels that hyperpolarizes the cell by increasing K+ 
conductance
Define the roles of (a) fast Na+-channels, 
(b) K+-channels, (c) voltage-gated Ca2+- 
channels, and (d) Na+-Ca2+ exchanger, 
in cardiac muscle contraction. 
• a) Fast Na+ channels: voltage-gated, open during phase 0, results in rapid 
depolarization. The upstrokes of all the ventricular muscle cell APs corresponds to 
the QRS complex 
• b) K+ channels: Two types; Transient outward type involved in phase 1 initial 
repolarization. Delayed rectifier type involved in phase 3 repolarization 
• c) Voltage-gated Ca2+ channels: contribute to slow inward, long-lasting current, 
plateau phase 2 
• d) Na+-Ca2+ exchanger: one mechanism to remove calcium from cells after it 
accumulates after action potentials. 3 Na+ ions in exchanged for 1 Ca2+ ion out.
Membrane Potentials 
• Chemical or Concentration Gradient: 
• created by concentration differences of ions 
• Electrostatic gradient: 
• positive ions will move to negative charge and vice-versa 
• Nernst Equation provides information about the membrane potential that 
is necessary to oppose the outward movement of a particular ion down 
its concentration gradient (the equilibrium potential of particular ion). 
• Ek=-61log[K+]inside/[K+]outside=-96 mV 
• But the membrane potential is also dependent on the conductance. This 
is where the semi-permeable characteristic of plasma membranes 
becomes important. 
• Em=g'K(EK)+g'Ca(ECa)+g'Na(ENa) 
• The equilibrium potential changes very little due to the relatively small 
changes in ion concentration. Thus the most important factor determining 
membrane potential is the differences in ion conductances.
List the major ions involved in establishing 
the cardiac membrane potential. 
• Na+, K+, and Ca2+ are the major ions that dictate membrane potential.
Define equilibrium potential and know its 
normal value for K+ and Na+ ions. 
• Equilibrium potential: the potential difference across the membrane 
required to maintain the concentration gradient across the membrane. 
• NOTE: Resting membrane potential for a cardiomyocyte is -90 mV 
(quite close to the equilibrium potential for K+). This is due to the fact 
that the membrane is much more permeable to K+ in a resting state.
Sketch a typical action potential in a 
pacemaker cell.
Sketch a typical action potential in a 
ventricular cell.
Describe the ion channels that contribute 
to each phase of the cardiac AP. 
• Nodal: Ca+ inflow, K+ Outflow, If current 
• AV node: Ca+ inflow, K+ Outflow, If current 
• Ventricular muscle: Na+ inflow, K+ outflow, Ca2+ inflow, K+ outflow 
• Purkinje cells: Na+ and Ca2+ inflow, K+ outflow, K+ outflow, If current
Describe the role of ion channels in AP 
generation and the effects of sympathetic 
and parasympathetic nerves on AP 
generation and HR. 
• Ion channels dictate the speed at which signals propagate through the heart. For 
instance during phase 0 in nonpacemaker cells, the rate of depolarization 
depends on the number of activated fast sodium channels. The more sodium 
channels the more rapidly the cell depolarizes. The faster the cell depolarizes, 
the faster the adjoining cell will depolarize and thus the more quickly the signal 
will propagate through the tissue. 
• Therefore conditions that decrease the availability of fast sodium channels (e.g., 
depolarization caused by cellular hypoxia), decreases the rate and magnitude of 
phase 0, thereby decreasing conduction velocity within the heart. 
• Sympathetic NS act on 훃1-adrenoceptors with norepinephrine to increase 
conduction velocity and thus heart rate. 
• Parasympathetic NS act on M2 receptors with acetylcholine to decrease 
conduction velocity and thus heart rate.
SPECTRUM OF CAD
Describe the spectrum of ischemic heart 
disease and its societal effects. 
• Ischemic heart disease is the leading cause of death in the world 
among men and women (7 million per year). 
• Coronary arteries cannot provide enough perfusion to keep up with 
myocardial demand. This results over time due to atherosclerotic 
narrowing of the arteries, along with superimposed degrees of plaque 
changes, thrombosis, and vasospasm. 
• CAD usually presents as an MI, Angina pectoris (most common), 
Chronic IHD with heart failure, and sudden cardiac death.
Compare Stable angina pectoris, 
Unstable angina pectoris, Acute MI.
What is the most common ECG finding 
during episodes of stable and unstable 
angina? 
• ST Depression
Define subendocardial versus transmural 
infarction and be able to differentiate the 
ECG segment changes 
• Subendocardial infarction involves small areas in the subendocardial 
wall of the left ventricle, ventricular septum, or papillary muscle ; ST 
depression. 
• Transmural infarction is associated with atherosclerosis in a major 
coronary artery; the infarct extends through the thickness of the heart 
muscle, resulting from nearly complete occlusions; ST elevation.
Identify ST elevation and pathologic Q 
waves and distinguish these findings 
associated with the anterior, lateral, or 
inferior walls.
Coronary Atheromatous Plaque 
• Plaque disruption is initiated by the release of substances from 
inflammatory cells within the fibrous cap. These weakened caps could 
then either rupture spontaneously or be due to a physical force such as 
in increase in intraluminal BP. 
• Following plaque rupture, a thrombus forms due to the activation of 
platelets (subendothelial collagen), the coagulation cascade, and 
narrowing of the vascular lumen (vasoconstrictors). 
• Dysfunctional endothelium  no release of vasodilators such as NO, 
which would normally oppose the effects of the vasoconstrictors and 
reduce platelet aggregation. 
• Destruction of myocytes in acute coronary syndromes quickly impairs 
ventricular contraction, which is seen as systolic dysfunction. 
• Ischemia/infarction will also impair diastolic relaxation, which reduces left 
ventricular compliance and leads to elevated filling pressure.
Identify the main difference between 
stunned and hibernating myocardium. 
• Stunned myocardium occurs when transient ischemia produces a 
prolonged (days to weeks) yet reversible period of contractile 
dysfunction. This may occur in UA (unstable angina) patients or 
surrounding areas of infarction. 
• Hibernating myocardium occurs when blood supply is chronically 
reduced, resulting in chronic contractile dysfunction. It promptly 
improves when adequate perfusion is restored.
Diagnostic Tests for CAD 
• ECG - look for ST segment elevation/depression and T wave changes 
- easy but may not “catch” episodes in outpatients 
• Stress test - provocative exercise or pharmacologic 
• Nuclear imaging - Te99 or Th201 perfusion studies 
• Coronary angiography 
• CT
Diabetics may be more likely to have 
silent angina
Describe the two main determinants of 
coronary blood flow. 
• The two main determinants of coronary blood flow are (diastolic) 
pressure and resistance (governed most strongly by a r^4 term).
Differentiate endothelial-dependent 
vasodilation from endothelial-independent 
vasodilation, and name one compound 
that works through each mechanism. 
• Endothelial-dependent vasodilation 
• NO 
• Prostacyclin 
• In atherosclerotic vessels, the endothelium is dysfunctional and 
fewer of these vasodilating molecules are released. 
• Endothelial-independent vasodilation 
• Adenosine 
• Lactate 
• Acetate 
• H+ 
• CO2
Myocardial O2 Supply and Demand
Name the primary mechanism by which 
coronary blood flow is maintained in the 
presence of moderate epicardial coronary 
artery stenosis. 
• Coronary arteries consist of both large, proximal epicardial segments 
and smaller, distal resistance vessels (arterioles). Atherosclerosis and 
narrowing almost always occur in the proximal vessels while the 
arterioles usually stay free of flow-limiting plaques. Thus, blood flow is 
maintained by the arterioles that can adjust their tone and dilate in 
response to metabolic needs. However, if the artery narrowing 
continues, the arterioles will eventually be unable to fully compensate.
Correlate the coronary artery that is 
occluded to the anatomic distribution of an 
acute myocardial infarction.
Complications 
Ventricular Wall Rupture: 3 to 7 days (associated with macrophages) 
The anterolateral wall at the midventricular level is the most common site. Gross photo shows tan yellow, soft 
infarct of 3-5 days duration with rupture of the ventricular wall. Due to excessive phagocytosis from infiltrating 
macrophages. Often fatal due to cardiac tamponade. 
Papillary muscle dysfunction: 2-7 days (associated with macrophages) 
Rupture of a papillary muscle may occur following an MI causing mitral regurgitation. More frequently, 
postinfarct mitral regurgitation results from ischemic dysfunction of a papillary muscle and underlying 
myocardium and later from papillary muscle fibrosis and shortening, or from ventricular dilation. 
Pericarditis: 2-3 days (associated with PMNs) 
A fibrinous pericarditis usually develops following a transmural infarct is a result of PMN infiltration in reaction 
to necrosis and myocardial inflammation. 
Mural thrombus: Within 10 days 
With any infarct, the combination of a local abnormality in contractility (causing stasis) and endocardial 
damage (creating a thrombogenic surface) can foster mural thrombosis and potentially thromboembolism. 
Ventricular Aneurysm: 
True aneurysms of the ventricular wall are bounded by myocardium that has become scarred. Aneurysms of 
the ventricular wall are a late complication (4-8 wks) of large transmural infarcts that experience early 
expansion. The thin scar tissue wall of an aneurysm paradoxically bulges during systole. Complications of 
ventricular aneurysms include mural thrombus, arrhythmias, and heart failure; rupture of the tough fibrotic wall 
is rarely a concern.
Describe the indications for coronary 
revascularization, including the need for 
coronary stenting 
• For angina patients, coronary revascularization is pursued if: 
• 1) Angina symptoms do not respond to anti-anginal therapy 
• 2) Drug therapy results in unacceptable side effects 
• 3) Patient is found to have high-risk coronary disease for which 
revascularization is known to improve survival 
• Persistent angina 
• Significant stenosis in 1 or 2 coronary arteries 
• Lower-risk patients with stenosis in all 3 CA
Limitations of Stenting 
• Restenosis: 
• Neointimal proliferation (migration of smooth muscle cells + ECM production) can 
occur over the stent, restenosing the vessel 
• Solution: cover the stent with an anti-proliferative medication that prevents 
neointimal proliferation. This decreases restenosis by 50%. 
• Unfortunately, this also slows endothelialization of the stent, resulting in a greater 
incidence of thrombosis if antiplatelet therapy is discontinued too early. 
• Give Prasugrel or Clopidogrel for 12 months for drug eluting stents and 1-2 months 
for bare metal stents 
• Thrombosis 
• Stent material is thrombogenic (promotes formation of clots) 
• Solution: Antiplatelet drug therapy 
• Give Prasugrel or Clopidogrel for 12 months for drug eluting stents and 1-2 months for 
bare metal stents
Coronary Bypass Surgery 
• More effective long-term relief of angina than PCI 
• Improved survival in patients with: 
• >50% left main stenosis 
• 3 vessel CAD, especially if LV contractile function is impaired 
• 2 vessel disease with >75% LAD stenosis 
• Diabetes patients with multiple vessels involved 
• More complete revascularization than PCI 
• Internal Mammary Artery vs. Veins 
• Veins are vulnerable to accelerated atherosclerosis (50% are occluded 
after 10 years) and lower 10 yr. patency rate (80%) 
• Arteries are have higher 10 yr. patency rates (90%) and are more 
resistant to atherosclerosis. 
• Specifically, the internal mammary artery appears to be fairly resistant 
to atherosclerosis 
• Limitations: stenosis of the grafted vessel due to atherosclerosis, surgical 
risk v benefits, comparison of benefits of CABG with PCI
CAD: PCI versus CABG 
PCI 
• Brief hospitalization 
• Less expensive 
• Minimally uncomfortable - 
percutaneous 
• Restenosis-9% 
• Stent thrombosis 
• Clopidogrel 
CABG 
• 5-7 days 
• More expensive 
• Painful 
• Usually definitive 
• Survival advantage 
• 3VD + reduced LVEF 
• Left main CAD
Exercise Testing 
• Positive if chest pain or ECG abnormalities are produced. 
• Exercise test considered markedly positive* if: 
• 1. Ischemic ECG changes develop in first 3 minutes or persist 5 
minutes after exercise stops 
• 2. Magnitude of ST segment depressions > 2 mm 
• 3. Systolic blood pressure abnormally falls during exercise (due to 
ischemia-induced impairment of contractile function) 
• 4. High-grade ventricular arrhythmias develop 
• 5. Patient cannot exercise for at least 2 minutes because of 
cardiopulmonary limitations
Coronary Angiography 
• Coronary Angiography 
• Most direct means of identifying coronary artery stenosis 
• Atherosclerotic lesions visualized radiographically following injection 
of radiopaque contrast material into artery 
• Procedure generally safe, but small risk of complication due to 
invasive nature 
• Reserved for patients whose angina symptoms do not respond to 
pharmacologic therapy, have an unstable presentation, or when 
results of noninvasive testing are so abnormal that severe CAD 
warranting revascularization is likely. 
• “Gold Standard” for CAD diagnosis, however… 
• Only provides anatomic information 
• Clinical significance of lesions depends on pathophysiologic consequences 
• Standard arteriography does not reveal composition of plaque or vulnerability to 
rupture
Coronary Blood Flow 
• In coronary arteries, most of blood flow to the myocardium occurs during diastole. 
• During systole: contraction of myocardium compresses ventricular microvasculature 
• Blood flow is reduced to the greatest extent within innermost regions of ventricular 
wall (subendocardium) where compressive forces are greatest 
• More susceptible to injury in ischemic events, CAD, reduced aortic pressure 
• Blood flow reaches peak in early diastole – where compressive forces removed 
• Aortic pressure during diastole thus is most 
crucial for perfusing coronaries 
• In left ventricle – mechanical forces affecting coronary flow are greatest 
• Due to the higher pressures associated 
• Right ventricle and the two atria show some effects of contraction and relaxation 
of blood flow 
• However, it’s much less apparent than the L ventricle
Explain how arterio-venous O2 difference 
and O2 extraction in the heart is unique 
when compared with other body organs. 
• Unlike most tissues, the heart cannot increase oxygen extraction on 
demand, because in its basal state it removes almost as much 
oxygen as possible from its blood supply. 
• So any additional oxygen requirement must be met by an increase in 
blood flow. 
• Autoregulation of coronary vascular resistance is most important 
mediator of this process 
• Factors regulating coronary vascular resistance: 
• Accumulation of local metabolites 
• Endothelium-derived substances 
• Neural innervation
Describe what is meant by coronary 
vascular reserve and the role of collateral 
blood vessels. 
• Coronary flow reserve is the maximum increase in blood flow through 
the coronary arteries above normal resting volume. 
• The CFR is reduced in coronary artery disease, i.e. there is reduced 
vasodilator reserve 
• When O2 delivery to heart limited by disease, collateral vessels arise 
through angiogenesis 
• Process stimulated by chronic stress (hypoxia, exercise training, etc.) 
• Collateralization increases myocardial blood supply by increasing the 
number of parallel vessels 
• Reduces vascular resistance within myocardium
List the four major coronary arteries and 
identify the structures they supply. 
• Left main coronary artery 
• Left anterior descending artery 
• Circumflex Artery 
• Right main coronary artery 
• Left coronary artery supply 
• Left ventricle 
• Left atrium 
• Interventricular septum 
• Right Coronary Artery supply 
• Right Atrium 
• Right ventricle 
• SA node 
• AV node 
• Interventricular septum
CHRONIC CORONARY 
SYNDROMES
Identify the factors that regulate 
myocardial oxygen consumption and 
myocardial oxygen delivery. 
• The myocardium has one of the highest O2 extraction ratio of all body organs. In 
a normal human being at rest the heart consumes 11 % of total body oxygen but 
receives only about 4 % of the cardiac output as coronary blood flow. 
• 
• Supply of oxygen depends upon: 
• · Oxygen content of blood (systemic oxygenation and hemoglobin) 
• · Coronary blood flow (perfusion pressure and coronary vascular resistance) 
• 
• Myocardial oxygen demand depends upon: 
• · Wall stress 
• · Heart rate 
• · Contractility 
• 
• O2 demand is increased by ↑ HR, ↑ heart contractility, ↑ preload, ↑ afterload, ↑ 
ejection time. 
• O2 supply is reduced by ↑ HR, ↑ preload, ↓ artery diameter (atherosclerosis).
Stable Angina 
• Angina is an imbalance between coronary blood flow and cardiac O2 
consumption leading to ischemia 
• Stable angina (classic angina, or angina of effort; most common 
form): 
• Ischemia is caused by stable coronary artery narrowing (atheromatous plaque) 
• Predictable pain on exertion or psychological stress. 
• Unchanged in severity, frequency, and duration over weeks to months
Pharmacologic treatments for angina
Diagram the neural pathway involved in 
anginal pain. 
• The myocardium is innervated by chemosensitive sensory afferent that are 
activated by products of hypoxia such as adenosine (a breakdown product of 
ATP), acidification (caused by anaerobiosis) and the release of autacoids such as 
serotonin and prostaglandins E. 
• The peripheral axons of these sensory afferents travel within the sympathetic 
chain. Their cell bodies are located in dorsal root ganglia (DRGs) at thoracic level. 
The central process contact spinal interneurons located in the dorsal horn of the 
spinal cord which in turn activate spino-reticular and spinothalamic pathways. The 
spino-reticular pathway activate the vasomotor center and therefore increases 
sympathetic tone to the heart potentially causing further ischemic damage by 
increasing oxygen demand. 
• Activation of the spinothalamic pathway causes anginal pain (except in people 
with silent angina). If pain occurs, this causes further increase in SNA via 
descending pathways through the medullary vasomotor center. 
• Anginal pain is referred to the neck shoulder and arm region because nociceptive 
afferents that originate from these regions of the bodies and cardiac nociceptors 
converge on the same spino-thalamic neurons.
Recall the mechanism whereby coronary 
blood flow is coupled to myocardial 
workload. 
• Increased contractility/HR increases ATP breakdown leading to 
increased local concentration of adenosine, stimulating vasodilation 
and increased coronary flow to meet the O2 demands with the 
increased myocardial workload.
Explain why pain, anxiety or exercise can 
exacerbate cardiac ischemia. 
• Pain, anxiety or exercise cause release of chemical mediators: NE, 5-HT that 
activate the sympathetic nervous system. 
• (1) The increase in sympathetic activity (β1 receptors) and the decrease in parasympathetic 
activity produce an increase in HR. 
• (2) The increase in sympathetic activity (β1 receptors) produces an increase in contractility and a 
resulting increase in SV. 
• Together, the increases in heart rate and stroke volume produce an increase in 
cardiac output. 
• This leads increased O2 demand will exacerbate cardiac ischemia. 
• Furthermore, pts with cardiac ischemia often have narrowed coronary arteries 
(atherosclerosis) so they have dysfunctional endothelium (less 
vasodilatory/antithrombogenic properties) along with increased resistance so that 
it is hard for the heart to keep up enough of an O2 supply with the increased 
workload. 
• Stimulation of alpha receptors by catecholemines released during stress, exercise 
and pain can lead to vasconstriction and without enough local metabolites like 
endothelial NO to offset that with vasodilatiion (dysfunctional endothelium) one 
could see less sympatholysis.
Identify the main goals of therapy for 
patients with stable angina. 
• Decrease frequency of anginal attacks 
• Prevent acute coronary syndromes 
• Prolong survival
MOA of nitroglycerin and isosorbide 
mononitrate on vascular smooth muscle 
and identify which blood vessels are 
preferentially targeted by low doses 
• Nitroglycerine is converted to Nitric oxide which activates guanylate cyclase leading to 
an increase in cGMP in smooth muscle. This leads to dephosphorylation of Myosin 
Light Chains which regulate the contractile state of smooth muscle and lead to 
vasodilation. 
• Nitroglycerin has a fast onset (2-5 minutes) with sublingual administration and a short 
duration (30 min). Low dose – relaxation of great veins 
• It is used for acute attacks of angina 
• Isosorbide Mononitrate: Organic nitrate that causes vasodilation through enzymatic 
conversion of sulfhydryl groups to nitric oxide 
• Slower onset, longer duration 
• Used for chronic treatment
Explain why nitroglycerin must be 
administered sublingually while isosorbide 
dinitrate or mononitrate is given orally. 
• Rapid onset of action and are useful as prophylaxis 
• Sublingual admin leads to fast absorption directly into the systemic 
circulation, thereby avoiding delay inherent to intestinal absorption 
• First pass metabolism (degradation) by the liver → nitroglycerin cannot 
be taken orally because it would be entirely degraded by the liver 
• Very lipid soluble compound that is well absorbed by the mucosa of the 
tongue and mouth 
• Spray more stable → in this formulation it stays active for years while 
tablets take up moisture which degrades the nitroglycerine within a 
month
Explain why NO donors and PDE5 
inhibitors (e.g. sildenalfil) should not be 
co-administered. 
• The combination of a PDE5 inhibitor (sildenafil (Viagra), etc) and 
nitrates  extreme hypotension 
• If PDE5 is inhibited cGMP cannot be converted to GMP, so much 
more cGMP is produced leading to extreme vasodilation
Nitrate tolerance 
• The main limitation to chronic nitrate therapy is the development of 
drug tolerance 
• Overcome this with: 
• 1) a nitrate-free interval for a few hrs (8-12 hrs each day) 
• 2) add drugs that reduce the requirement for nitrates (such as B-blockers 
or Ca++ channel blockers) 
• Mechanism theories include: 
• 1) Sulfhydryl hypothesis  depletion of SH groups need for 
conversion to NO 
• 2) Neurohormonal hypothesis  reflex increase in vasoconstrictor 
hormones (NE, tissue RAS, endothelin) 
• 3) Free radical hypothesis  free radicals destroy NO
Explain the rationale for the potential 
benefit of combining a beta-blocker with a 
nitrate in treating stable angina.
Statins 
Mechanism Competitively inhibit HMG-CoA reductase: 
(1) Decreases intracellular cholesterol  induces SREBP  
increases expression of LDL-R 
(2) VLDL and IDL are cleared more rapidly due to cross-recognition 
with hepatic LDL-R 
(3) Hepatic VLDL production falls due to reduced cholesterol 
availability  reduced LDL and triglycerides 
Modify platelets and endothelium (e.g., enhanced NO 
synthesis) 
Suppress inflammation 
Effects Decreases LDL 18-55% 
Decreases TG 7-30% 
Increases HDL 5-15% (unclear mechanism) 
Side Effects Myopathy (increased w/ niacin, fibrates), hepatotoxicity, drug 
interactions (CYP3A4 inhibition: macrolides, azoles, HIV 
protease inhibitors)
Niacin 
Mechanism Decreases lipolysis in adipose tissue  less FAs available for 
TG synthesis in liver 
Decreases VLDL synthesis, so less LDL 
Increases HDL by decreasing hepatic removal of HDL 
Effects Decreases LDL 5-25% 
Increases HDL 15-30% 
Decreases TGs 20-50% 
Side Effects Cutaneous flushing (due to prostaglandins; take aspirin), GI 
(nausea, PUD), hepatotoxicity, insulin resistance and 
hyperglycemia (caution w/ diabetics), gout (raises serum uric 
acid levels), myopathy (increases w/ statin)
Fibrates 
Mechanism Activate PPARα-RXR  
(1) Enhanced oxidation of FAs in liver and muscle  
decreased TG levels  decreased VLDL 
(2) Increased expression of LPL 
(3) Increased rate of HDL-mediated reverse cholesterol 
transport (due to apo AI transcription) 
Effects Decreases LDL 5-20% 
Increases HDL 10-20% 
Decreases TGs 20-50% 
*Larger decreases in TGs and increases in HDL than statins. 
Side Effects GI (dyspepsia, abdominal pain, diarrhea), cholesterol 
gallstones, myopathy (increased w/ liver and kidney 
dysfunction; worse w/ statins), augment effects of oral 
hypoglycemic drugs (avoid in diabetes)
List the major side effects of antianginal 
medications, including which drugs when 
combined have an increased risk of SEs 
• Nitrates: headache, lightheadedness, hypoTN, palpitations, nausea, dizziness, reflex sinus 
tachycardia (BP decrease causes heart to compensate and beat faster). 
• DO NOT ADMINISTER with PDE5 inhibitors. 
• B-blockers: bronchospasm  AVOID in pts with COPD, reduced HR  contraindicated in pts with 
bradycardia, fatigue, sexual dysfunction, may worsen diabetic control and can mask tachycardia and 
other signs that indicate hypoglycemia  *** Diabetic pts 
• Calcium channel blockers: associated with an increased incidence of MI and mortality, Headache, 
flushing, decreased LV contraction (esp with Verapamil and Diltiazem) , pedal edema (esp with 
Nifedipine and Diltiazem), constipation (esp with Verapamil) 
• Combining a B-blocker with a nondihydropyridine Ca++ channel blocker (Verapamil or Diltiazem): 
negative chronotropic (“changing HR”) effect that can cause excessive bradycardia, combined with 
a negative inotropic effect, could precipitate heart failure in pts with LV contractile dysfunction 
• Ranolazine: dizziness, headache, constipation, nausea
Describe the mechanism of Prinzmetal 
(variant) angina 
• Prinzmetal’s (variant) angina is ischemia due to focal coronary artery 
spasm. 
• Mechanism: Patient has intense vasospasm. → Reduced coronary 
oxygen supply → Prinzmetal’s Angina. 
• The cause of the vasospasm is unknown, but it is thought that it may be 
caused by increased sympathetic activity + endothelial dysfunction. 
• Patient presentation: Typical anginal discomfort, usually at rest rather 
than upon exertion. The pain is often very severe. 
• ECG findings: ST segment elevations during the intense vasospasm. ST 
segment elevation signifies injury. 
• Good antianginal drugs for this type of angina: Verapamil & Diltiazem 
• Beta Blockers are contraindicated as they may worsen the condition
Describe the typical ECG findings during 
an episode of stable angina. 
• ST segment and T wave changes 
• Transient horizontal or downsloping ST segment 
depressions 
• T wave flattening or inversions 
• Occasionally ST segment elevations are seen, 
suggesting more severe transmural myocardial 
ischemia 
• In contrast to an acute MI, ST deviations 
caused by angina quickly normalize with 
resolution of the patient’s symptoms.
Describe the findings of a positive 
exercise ECG treadmill stress test. 
• The test is considered positive if the patient’s typical chest discomfort 
is reproduced or if ECG abnormalities consistent with ischemia 
develop (ex: >1 mm horizontal or downsloping ST segment 
depressions) 
• The test is considered markedly positive if one or more of the 
following signs of ischemic heart disease occur: 
• Ischemic ECG changes develop in the first 3 minutes of exercise OR persist 5 
minutes after exercise has stopped 
• The magnitude of ST segment depressions is >2 mm 
• The systolic blood pressure abnormally falls during exercise (i.e. resulting from 
ischemia induced impairment of contractile function) 
• High grade ventricular arrhythmias develop 
• The patient cannot exercise for at least 2 minutes because of cardiopulmonary 
limitations
Recall the effect of a 60% epicardial 
coronary artery stenosis on resting 
coronary blood flow versus its effect on 
maximal coronary flow. 
• This is pointing to the fact that if 
an epicardial coronary artery is 
stenosed to a level of 60%, this 
will limit the maximum amount 
of blood that can flow through a 
coronary artery (and therefore 
may lead to angina when you 
are exercising or exerting 
yourself), but it will have no 
effect on the resting coronary 
blood flow, which is much less.
Explain why nuclear cardiac imaging and 
echocardiography are sometimes 
performed in conjunction with exercise 
stress testing. 
• Some patients have baseline ST segment abnormalities or T wave 
abnormalities (particularly LVH with strain). For these patients, the 
ECG findings provided by the exercise stress test aren’t very useful. 
• Also, the exercise stress test can yield ambiguous results - this is 
especially of concern when there is a high clinical suspicion of 
ischemic heart disease. 
• Exercise stress testing in conjunction with nuclear cardiac imaging or 
echocardiography increases sensitivity and specificity.
Explain the most common reasons that 
percutaneous revascularization (PCI) is 
offered to patients with stable angina. 
• Many patients with stable angina can be managed with 
pharmacologic therapy alone. 
• PCI is offered to patients with stable angina if: 
• The patient’s symptoms of angina do not respond adequately to 
antianginal drug therapy 
• Unacceptable side effects of medications occur 
• The patient has high risk coronary disease for which 
revascularization is known to improve survival
Name one criterion by which patients are 
selected for CABG instead of PCI. 
• CABG is good for: 
• >50% left main stenosis 
• 3 vessel CAD, especially if LV contractile function is impaired 
• 2 vessel disease with >75% LAD stenosis 
• Diabetes patients with multiple vessels involved 
• PCI is good for: 
• Patients with persistent episodes of angina and significant stenoses in one to two 
coronary arteries 
• Some lower risk patients with three-vessel disease
ACUTE CORONARY 
SYNDROMES
Describe the pathophysiologic events that 
change a stable atherosclerotic plaque 
into the unstable plaque of ACS 
• Increasing size and protrusion of lipid core  mechanical stress 
focused on plaque border 
• Local accumulation of foam cells and T lymphocytes releasing MMPs 
 increases degradation of ECM. 
• Thin fibrous cap  vulnerability to rupture 
• Rupture of plaque  exposure of procoagulants  thrombosis 
• Causes occlusion and infarction 
• Endothelial dysfunction prevents release of endogenous vasodilators 
(NO, prostacyclin) which also normally inhibit platelets, thus impairing 
protective mechanisms against thrombosis.
Compare and contrast the 
pathophysiologic and clinical features of 
unstable angina, non-STEMI and STEMI.
Recognize the non-atherosclerotic causes 
of an acute coronary syndrome.
Describe the functional alterations 
impairing contractility and compliance. 
• Systolic dysfunction - destruction of functional myocardial cells 
leading to impaired ventricular contraction 
• Hypokinetic: a localized region of reduced contraction 
• Akinetic - a segment that does not contract at all 
• Dyskinetic - a segment that bulges outward during contraction of the remaining 
functional portions of the ventricle 
• Diastolic dysfunction - compromise of the left ventricle when 
ischemia/infarction causes elevated ventricular filling pressures
Define and distinguish the terms “stunned 
myocardium”, “ischemic preconditioning” 
and “infarct expansion” 
• Stunned myocardium - prolonged but reversible period of contractile dysfunction 
after a period of transient ischemia. The tissue prolongs systolic dysfunction even 
after restoration of blood flow. Contractile force is regained days to weeks later. If 
the tissue is simply stunned rather than necrotic, its function will recover. 
• Ischemic preconditioning - brief ischemic insults to a region of myocardium that 
make that region more resistant to subsequent episodes. Thus, patients who 
have an MI after recent anginal pain often have less morbidity and mortality than 
those with an “out-of-the-blue” MI. The conditioning may be triggered by 
substances released during ischemia like adenosine and bradykinin. 
• Infarct expansion - in an early post-MI period, the affected ventricular segment 
enlarges without additional myocyte necrosis - occurs by thinning and dilatation of 
the necrotic zone from “slippage” between muscle fibers. 
• The increase in ventricular size 1) augments wall stress, 2) impairs systolic contractile function, 
and 3) increases the likelihood of aneurysm formation
Cellular Changes of Acute MI 
• Occluded coronary vessel  falling O2 levels  switch from aerobic to anaerobic 
metabolism  lactic acid accumulation  lowered pH (Metabolic Acidosis) 
• The decrease in high-energy phosphates like ATP interfere with Na+/K+ ATPase 
 elevation in intracellular Na+ and extracellular K+ 
• Rising intracellular Na+  cellular edema 
• Membrane leakage and rising extracellular K+  altered transmembrane 
electrical potentials and predisposition to arrhythmias 
• Intracellular Ca2+ accumulation activates degradative lipases and proteases and 
contributes to final pathway of cell destruction. 
• Metabolic changes decrease function within 2 minutes of an occlusive thrombosis. 
• Without intervention, irreversible cell injury ensues within 20 minutes 
• Proteolytic enzymes leak across membrane and damage myocardium 
• Release of macromolecules into circulation (Troponin @ 4 hours)
Compare and contrast the use and time 
course of troponin and CPK-MB in 
diagnosis an acute MI 
• Troponins 
• The specific troponins used for the diagnosis of MI are cTnI and cTnT, because 
these are the cardiac forms of troponin I and are also structurally unique, and thus 
easier to assay. 
• These markers tend to increase 3-4 hours after onset of discomfort, peak 
between 18-36 hrs and may be present for up to 2 weeks. 
• CPK-MB 
• Isoenzyme of creatine kinase that exists in the heart - CPK-MB. 
• The measurement for CPK-MB is calculated by the ratio of CPK-MB: total CPK. 
• Values of >2.5% usually indicative of cardiac injury 
• Levels of CPK-MB rises between 3-8 hrs following infarction, peaks at 24 hrs and 
goes back to normal after 48-72 hrs. 
• This makes it a useful indicator for REINFARCTS. 
• Neither of these markers are good for early diagnosis of MI, since they take a few 
hours to peak. In early situations, ECG and history are most important.
Do not use Fibrinolytic treatment 
regimens for NSTEMI
Name at least two fibrinolytic agents and 
explain the benefits, limitations and major 
risks of thrombolytic therapy. 
• Alteplase, tPA 
• Reteplase, rPA 
• Tenecteplase - TNK-tPA 
• Very effective in lysing the intracoronary thrombi found in STEMI. 
Patients who receive FT quickly (within 2 hours of onset of symptoms) 
have half the rate of mortality of STEMI pts who get it after 6 hrs 
• Fibrinolytic therapy does not benefit patients suffering from UA or 
NSTEMI. 
• Bleeding is the most common complication of fibrinolytic therapy, patients 
who require effective fibrin clotting are contraindicated for this therapy. 
That includes post-op patients, those with a bleeding disorder or recent 
stroke.
Define the term “primary percutaneous 
coronary intervention” and explain the 
benefits, limitations and major risks 
• Angioplasty + stenting of the vessel 
• Medications given during PCI: Aspirin, Heparin, IV GP IIb/IIIa receptor antagonist 
• *may substitute Direct Thrombin Inhibitor (e.g. Bivalirudin) for Heparin + GP IIb/IIIa antagonist combo 
• If receiving a stent → Oral Thienopyridines (e.g. Clopidogrel) given to reduce risk of 
ischemic complications & stent thrombosis 
• Clopidogrel or Prasugrel given for >12 months after stent placed 
• Benefits: 
• Treat patients with contraindications to Fibrinolytic therapy or unlikely to do well with 
fibrinolysis (e.g. late presentation to hospital - more than 3 hrs with symptoms, in 
cardiogenic shock) 
• Treat patients initially treated with fibrinolytic therapy without adequate response (e.g. 
ST segment elevations) 
• In comparison to Fibrinolytic Therapy: 
• Greater survival & Lower rates of reinfarction and bleeding 
• Preferred method IF performed by experienced operator within 90 mins of arriving
TIMI Risk Score 
• Patients with “most concerning clinical features” → 
identified by risk assessment algorithms (higher scores: ≥ 
3)TI 
• Age > 65 years old 
• ≥ 3 Risk factors for coronary artery disease 
• Known Coronary Stenosis of ≥ 50% by prior angiography 
• ST segment deviations on ECG at presentation 
• ≥ 2 anginal episodes in prior 24 hrs 
• Use of Aspirin in 7 days prior 
• Elevated serum troponin or CK-MB
Identify the most important predictor of 
post-MI outcome, and describe how to 
risk-stratify a patient after acutely treating 
their myocardial infarction. 
• Most important predictor of post-MI outcome - LV Dysfunction 
• Identify patients at high risk for complications: 
• Exercise treadmill testing 
• Attention to underlying cardiac factors 
• Smoking 
• Hypertension 
• Diabetes 
• LV ejection fraction of less than or equal to 30% after MI  high risk 
of sudden cardiac death 
• Prophylactic placement of implantable cardioverter-defibrillator recommended
Ejection Fraction of <30% after MI is high 
risk for sudden cardiac death and 
suggests prophylactic placement of ICD.
INTRO TO EKG
Know the electrode placements and 
polarities for a 12‐lead electrocardiogram
Frontal Plane Electrodes
Standard values for EKG print out
Correlate tracing to electrical state of heart
Systematic Approach 
• Rate 
• Rhythm 
• Axis 
• Intervals 
• Hypertrophy 
• Ischemia 
• Special Changes
RATE
A rate of 60-100 is normal <60 is 
bradycardia and >100 is tachycardia
Method 1 for Determining Rate
Method 2 for Determining Rate
Method 3 for Determining Rate
RHYTHM
Criteria for Normal Rhythm 
• A P wave morphology P wave (atrial contraction) precedes every 
QRS complex 
• A QRS complex follows every P wave 
• The rhythm is regular, but varies slightly during respirations 
• The rate ranges between 60 and 100 beats per minute 
• The P waves maximum height at 2.5 mm in II and/or III 
• The P wave is positive in I and II, and biphasic in V1
Is there a QRS after every P? 
• NO 
• If rate < 100: 
• a) 2 block type I ("Wenkebach," gradually lengthening PR until one 
beat is dropped) 
• b) 2 block type II (dropped beat without change in PR), or 
• c) 3 AV block (no correlation between P and QRS) 
• If rate >100: 
• atrial or nodal tachycardia (SVT) or atrial flutter, both with block.
Is there a P before every QRS? 
• NO 
• a) if a single slow beat: escape beat 
• atrial if different P 
• nodal if no P 
• ventricular if QRS>0.12 
• b) if a slow rhythm: escape rhythm 
• nodal if no P at rate 50-60 
• ventricular if QRS>0.12 & rate <40 
• c) if a single fast beat: premature beat 
• PAC if different P 
• PJC if no P 
• PVC if QRS>0.12
Tachycardic 
• a) if wide (>0.12) and rate >120 then ventricular tachycardia until 
proven otherwise 
• b) if narrow and regular then atrial (with preceding P) or AV nodal (no 
P) tachycardia 
• c) if irregularly irregular then either 
• 1) atrial fibrillation (no P waves and coarse baseline) 
• 2) multifocal atrial tachycardia (MAT, three different P wave morphologies).
AXIS
Define mean electrical vector (axis) of the 
heart and give the normal range.
Determine the mean electrical axis from 
knowledge of the magnitude of the QRS 
complex in the standard limb leads. 
• Inspect limb leads and 
determine QRS that is most 
isoelectric, the mean axis is 
perpendicular to that lead 
• Inspect the lead that is 
perpendicular to the 
isoelectric complex, if the 
QRS is primarily upward, 
then the mean axis points 
towards the (+) pole of the 
lead.
Quadrant Approach
INTERVALS
Intervals 
• PR 
• Measure of the health of the AV node and bundle of His 
• Normal: < 0.2 s (1 big box) 
• Pathology: Prolonged interval: AV blocks 1,2,3 
• QRS 
• Measure of the health of the His-Purkinje system 
• Normal: < 0.12 s (3 small boxes) 
• Pathology: Conduction delay: LBBB,RBBB, fascicular blocks 
• QT 
• Measure of repolarization 
• Normal: < 0.45 s 
• Pathology: Long QT syndrome, electrolyte imbalance, ischemia
First Degree AV Block 
• Prolonged PQ Interval >.2s
Second Degree AV Block-Wenkebach 
• In second degree AV block type I, the PQ interval prolongs from beat 
to beat up until the drop-out of one QRS complex. The characteristics 
of a Wenkebach block: 
• QRS complexes cluster 
• The PQ interval prolongs every consecutive beat 
• The PQ interval that follows upon a dropped beat is the shortest. 
• The amount of block decreases during exercise
Second Degree AV Block – Mobitz II 
• In second degree AV block type II, beats are dropped irregularly 
without PQ interval prolongation. 
• As the drop out of beats is irregular, no clustering of QRS complexes can be seen 
as in second degree block type I. 
• Second degree AV block type II marks the starting of trouble and is a class I 
pacemaker indication
Third Degree AV Block 
• Third degree AV block is synonymous to total block: absence of 
atrioventricular conduction. The P-waves and QRS complexes have 
no temporal relationship: AV dissociation. The ventricular rhythm can 
be nodal, idioventricular or absent. 
• During third degree AV block the blood supply to the brain can insufficient, leading to 
loss of consciousness. Adams Stokes attacks attacks are attacks of syncope or pre-syncope 
in the setting of third degree AV block.
QRS Interval 
• Measure QRS interval, if > 0.12 then Look at V1 for LBBB and RBBB 
• Left bundle branch block (downgoing) 
• Right bundle branch block with (bunny ears and upgoing)
LBBB
RBBB
LAFB 
• Left Axis Deviation 
• qR in the lateral leads 
• rS in the inferior leads 
• Mild QRS widening
LPFB 
• Right Axis Deviation 
• rS in lateral leads 
• qR in inferior leads 
• Mild or no QRS widening
QT Interval 
• Bazett’s Formula 
• QTc = [QT Interval] / √[R-R interval] 
• Important due to R on T risks 
• Acquired 
• Medications 
• Electrolyte abnormalities 
• Ischemia 
• Hypothermia 
• Genetic 
• Sodium Channel abnormalities 
• Potassium Channel abnormalities 
• High risk for ventricular fibrillation
Brugada Syndrome 
• ST elevation ≥2 mm and a coved type ST segment followed by a 
negative T wave. This morphology must be present in >1 right 
precordial lead (V1-V3).
Wolff Parkinson White 
• Short PR interval with delta wave
Digoxin Intoxication
HYPERTROPHY
Differentiate left atrial enlargement from 
right atrial enlargement on an ECG 
• Right Atrial Enlargement 
• Lead II – P wave > 2.5 mm 
• Lead V1 – Biphasic p wave with upright deflection larger 
• Left Atrial Enlargement 
• Lead II – The P wave is broader 
• P mitrale 
• Lead V1 – Biphasic p wave with terminal component larger
Atrial Enlargement
Right Atrial Enlargement
Left Atrial Enlargement
LEADS II AND V1 ARE MOST PARALLEL 
TO ATRIAL DEPOLARIZATION BEST 
AREA TO VIEW P WAVE
ECG of ventricular hypertrophy
Left Ventricular 
Hypertrophy 
• R in V5 or V6 + S in V1 >35 mm
Look for ST depression STRAIN
Right Ventricular 
Hypertrophy 
• QRS duration < 120ms 
• Right heart axis (> 110 degrees) 
• Dominant R wave in V1
INFARCTION
Infarction 
• Ischemia: inverted T waves or ST depression 
• Injury: ST elevation 
• Necrosis: Pathologic Q waves
Myocardial Ischemia 
• ST depression and T-wave changes. 
• New horizontal or down-sloping ST depression >0.05 mV in two contiguous leads; 
and/or T inversion ≥0.1 mV in two contiguous leads with prominent R-wave or R/S 
ratio ≥ 1 
• ST elevation 
• New ST elevation at the J-point in two contiguous leads with the cut-off points: ≥0.2 
mV in men or ≥ 0.15 mV in women in leads V2–V3 and/or ≥ 0.1 mV in other leads.
Myocardial Infarction
Myocardial Necrosis 
• Pathologic Q waves – irreversible injury 
• Localize based upon which arteries are involved 
• Any Q wave > 1 small box in duration and more than 1/3 the height of 
the R wave in 2 contiguous leads.
Predict which coronary artery is affected in 
a patient experiencing an acute 
myocardial infarction.
Anterior Hemiblock 
• LAD occlusion 
• Normal or slightly widened QRS 
• Q1S3
The only upside down QRS complex 
allowed is in AVR. Do not assess Q waves 
in AVR.
ST segment depression in NSTEMI is 
nonlocalizing to specific arteries
Be careful about diagnosing an infarct in 
the presence of LBBB.
THE CARDIAC CYCLE
Outline the phases of the cardiac cycle 
and describe which phases demarcate 
systole and diastole. 
• There are 7 phases in the cardiac cycle. 
• Systole: phases 2-4 
• ventricular contraction and ejection 
• Diastole: phases 5-7 and 1 
• ventricular relaxation and filling
The Cardiac Cycle 
• 1. During diastole the mitral valve is open so LA and LV have equal 
pressure 
• 2. Late diastole, LA contraction causes small rise in pressure (a wave) 
• 3. Systolic contraction, LV pressure rises and MV closes when LV 
exceeds LA pressure (S1) 
• 4. When LV pressure exceeds aortic pressure, AV valve opens (silent) 
• 5. Ventricle relaxes, pressure drops below aorta, AV closes (S2) 
• 6. LV pressure falls below and MV opens (silent)
Atrial Waveform 
• a wave: Increase due to atrial contraction. 
• x descent: Fall in atrial pressure due to end of atrial contraction. 
• c wave: Increase due to bulging of AV valves back into atrial 
chambers. 
• x’ descent: Fall in atrial pressure after ‘c wave’ due to rapid ventricular 
ejection. 
• v wave: Peak due to continuing venous return just prior to AV valves 
opening. 
• y descent: Rapid fall in atrial pressure after ‘v wave’ due to opening of 
AV valves.
Systole occurs approximately between 
the S1 and S2 heart sounds
Derive the stroke volume and left 
ventricular ejection fraction from the left 
ventricular end-systolic and end-diastolic 
volumes. 
• SV= LVEDV-LVESV 
• EF = SV/LVEDV 
• Normal ≥ 55% 
• The left ventricular ejection fraction is the percentage of blood that 
leaves the left ventricle with each contraction.
Diagram the timing of the s1, s2, s3, and 
s4 heart sounds to the left ventricular 
pressure curves and identify the 
mechanical events that cause each 
sound. 
• S1 = Mitral valve closes 
• S2 = Aortic valve closes 
• S3 = Mitral valve opens. This can be normal in children and pregnant 
women but is pathological in adults. Systolic defect associated with 
ventricular dilation. 
• S4 = Pathological. Blood is being forced against a stiff ventricle. Ex: 
Left ventricular hypertrophy. Diastolic defect.
Describe the relative contribution of 
passive and active left ventricular filling 
and the effects of heart rate and 
sympathetic activation on this ratio. 
• Active ventricular filling is associated with atrial contraction, while passive ventricular 
filling depends on venous return and occurs before atrial contraction. 
• At Rest: Active filling accounts for approximately 10% of total left ventricular filling. 
• At High Heart Rates (Exercise): Active filling accounts for up to 40% of left ventricular 
filling. This increased contribution results from two factors: 
• Increased heart rate leads to shortened periods of diastolic filling → Reduced amount of blood 
entering the ventricle during passive filling. 
• Sympathetic nerve activation increases the force of atrial contraction → Increased amount of 
blood entering the ventricle during active filling. 
• This phenomenon is known as “atrial kick” 
• Clinical Aside: In atrial fibrillation, atrial contraction does not contribute to ventricular 
filling. This leads to inadequate filling that is exacerbated during physical activity.
State the mean right and left atrial 
pressures and peak and mean right and 
left ventricular pressures 
• Mean Right Atrial Pressure: 4 mmHg (average) 
• Mean Right Ventricular Pressure: 25 mmHg (systolic) and 4 mmHg 
(diastolic) 
• Mean Left Atrial Pressure: 8 mmHg (average) 
• Mean Left Ventricular Pressure: 120 mmHg (systolic) and 8 mmHg 
(diastolic) 
• The left heart has higher average pressures because the left ventricle 
must eject blood into the entire systemic circulation. The right heart 
has lower average pressures because it is responsible for ejecting 
blood only to the lungs.
Dicrotic Notch in Pressure Tracing 
• A very brief and transient increase in aortic pressure that corresponds 
with closure of the aortic valve at the conclusion of systole.
Define cardiac output and cardiac index 
and describe their relationship 
• Cardiac Output: CO = SV X HR 
• Cardiac Index (CI) = CO / BSA 
• Cardiac index is a variation of cardiac output that normalizes for the 
size of the individual.
Describe the relationship between SV and 
HR and their relative influence on cardiac 
output. 
• CO = HR X SV is the basic relationship. However, since changes in 
heart rate can affect stroke volume, changes in heart rate do not 
correspond to an exactly proportional change in cardiac output. 
• Examples: 
• As heart rate increases through pacemaker stimulation, ventricles 
have less time to fill with blood during diastole. Less ventricular filling 
corresponds with a decreased stroke volume. 
• When heart rate increases by 2X due to pacemaker stimulation alone, 
cardiac output increases less than 2X. 
• When heart rate increases by 2X due to exercise, cardiac output 
increases more than 2X.
STEMI TBL
Know the symptoms and signs acute 
coronary syndrome 
• Retrosternal pressure radiating to neck, jaw or left shoulder and arm 
(C7-T4); more severe and lasts longer than previous anginal attacks 
• Sympathetic response: Diaphoresis, tachycardia, cool, clammy skin 
• Systolic dysfunction: dyspnea 
• Ventricular noncompliance: S4 and S3 heart sounds 
• Inflammation: Fever 
• Serum Markers: Increased troponin and CK-MB 
• ECG: ST depression or elevation, inverted T wave, Q wave
ECG of Unstable Angina and NSTEMI
ECG findings of an acute MI
Localize the site of an infarction and know 
the likely infarct related artery based on an 
ECG.
Ventricular Fibrillation and Tachycardia 
• Rapid, disorganized electrical activity of the ventricles 
• Most fatal before arrival at hospital 
• If present 48 hours after MI, then typically reflects severe left 
ventricular dysfunction and is associated with high mortality rates 
• If it presents <48 after MI, prognosis is much better and often due to 
transient electrical instability
Atrial Fibrillation 
• Result from atrial ischemia or atrial distension second to LV failure
Sinus Tachycardia and Bradycardia 
• Bradycardia: due to excessive vagal stimulation or SA nodal ischemia 
in the setting of inferior wall MI 
• Tachycardia: due to pain, anxiety, heart failure, drug administration or 
intravascular volume depletion 
• Can exacerbate ischemia
Complete Heart Block 
• May result from ischemia or necrosis of conduction tracts 
• May develop transiently from increased vagal tone
Explain the difference between ST 
elevation MI (STEMI) and non STEMI and 
discuss why a timely diagnosis is most 
important for a STEMI. 
• A STEMI involves the complete occlusion of one of the coronary arteries 
and leads to severe ischemia 
• ST elevation localizes on ECG based on which artery is involved 
• A timely diagnosis is extremely important for as irreversible damage to 
myocytes begins to occur after about 20 minutes. 
• Changes seen later in ECG such as inversion of the T wave and 
abnormal Q waves can also be avoided following successful treatment if 
MI is recognized.
Fibrinolysis vs. primary PCI for treatment 
of an acute MI. 
• NSTEMI: With a NSTEMI, fibronolysis is never used as patients do not benefit 
from this therapy. The decision whether to proceed with PCI is based upon a 
patient’s TIMI score. An early invasive strategy is recommended in patients with 
higher scores (≥3). If an early invasive approach is adopted, the patient should 
undergo angiography within 24 hours. 
• STEMI: In contrast to UA and NSTEMI, the culprit artery in STEMI is typically 
completely occluded, and therefore, the major focus of acute treatment is to 
achieve prompt reperfusion of the jeopardized myocardium using either fibrinolytic 
drugs or percutaneous coronary mechanical revascularization. 
• Primary PCI is usually the preferred reperfusion approach in acute STEMI, if 
the procedure can be performed by an experienced operator within 90 
minutes of hospital presentation. 
• In addition, primary PCI is preferred for patients who have contraindications to 
fibrinolytic therapy or are unlikely to do well with fibrinolysis, including those who 
present late (>3 hours from symptom onset to hospital arrival) or are in 
cardiogenic shock. Furthermore, “rescue” PCI is recommended for patients 
initially treated with fibrinolytic therapy who do not demonstrate an adequate 
response.
Describe the rationale and the indication 
for adjunctive therapies for the 
management of the ACS 
• Focus of treatment for STEMI, UA and NSTEMI consists of anti-ischemic 
medications to restore the balance between myocardial 
oxygen supply and demand and anti-thrombotic therapy aimed at 
preventing further growth, and facilitating resolution of the underlying 
occlusive coronary thrombus. 
• Beta blocker: lower heart rate 
• Propanolol 
• Metoprolol, Atenolol, esmolol, acebutolol 
• Aspirin/Clopidogrel: antiplatelet 
• Aspirin 
• Clopidogrel 
• Prasugrel 
• Ticagrelor 
• ACE Inhibitors: reduces LV remodeling 
• Statins: treats atherosclerosis
Define "cardiogenic shock" and explain 
the clinical manifestations of this 
syndrome. 
• Cardiogenic shock is a condition of severely decreased cardiac 
output and hypotension (systolic blood pressure < 90 mm Hg) with 
inadequate perfusion of peripheral tissues that develops when 
>40% of the LV mass has infarcted (after MI). 
• Chest pain/pressure, tachypnea, tachycardia, weak pulse, skin that is 
pale/blotchy/sweaty, lightheaded, disoriented, syncope, coma, 
decreased urination.
List the four types of "non-cardiogenic 
shock" and be able to explain the 
differences from cardiogenic shock. 
• 1. Hypovolemic shock accompanies significant hemorrhage, or fluid loss from severe burns, 
chronic diarrhea, or prolonged vomiting. The direct consequence of hypovolemia is 
inadequate cardiac filling and reduced stroke volume  reduced CO. 
• 2. Anaphylactic shock is a result of an allergic reaction. This immediate hypersensitivity 
reaction is mediated by histamine, prostaglandins, leukotrienes, bradykinin that results in 
substantial arteriolar vasodilation, increases in microvascular permeability, and loss of 
peripheral venous tone. These combine to reduce both total peripheral resistance and 
cardiac output. 
• 3. Septic shock is also caused by profound vasodilation but specifically from substances 
released into the circulating blood by infective agents 
• 4. Neurogenic shock is produced by loss of vascular tone due to inhibition of the normal tonic 
activity of the sympathetic vasoconstrictor nerves and often occurs with deep general 
anesthesia or in reflex response to deep pain associated with traumatic injuries. It may also 
be accompanied by an increase in vagal activity, which significantly slows the cardiac beating 
rate. This type of shock is often referred to a vasovagal syncope.
Explain the cardiovascular alterations 
occurring in shock (both compensatory 
and decompensatory). 
• Compensatory: Increased sympathetic and decreased 
parasympathetic response. 
• Rapid, shallow breathing  increase venous return 
• Renin, vasopressin, epinephrine release  increase vasoconstriction, 
• Glycogenolysis  fluid shift 
• Decreased organ blood flow (particularly kidneys) 
• Decompensatory : Reduced organ blood flow  Drive to reduce 
arterial pressure  positive feedback cycle 
• These decompensatory mechanisms are compounded by a reduction in 
sympathetic drive and a change from vasoconstriction to vasodilation with 
a further lowering of blood pressure. Can lead rapidly to death.
Outline the initial management in the 
treatment of cardiogenic shock. 
• Early cardiac catheterization and revascularization can improve 
prognosis prior to occurrence. 
• Tx of shock: 
• IV inotropic agents Dobutamine, Dopamine 
• Increase contractile force 
• Arterial vasodilators Hydralazine and Ca2+ Channel Blockers 
• Once BP improves to reduce resistance to LV contraction 
• Placement of intra-aortic balloon pump into aorta via femoral artery 
• Percutaneous Left Ventricular Assist Device (LVAD)
Main difference of cardiogenic shock vs. 
noncardiogenic shock physiology is that 
cardiogenic shock causes decreased CO 
by DIRECTLY IMPACTING THE 
CONTRACTILITY OF THE 
MYOCARDIUM while the other forms of 
shock indirectly impact CO.
Describe how the susceptibility to 
infarction varies between the myocardial 
layers. 
• Transmural infarcts 
• Span the entire thickness of the myocardium 
• Result from total, prolonged occlusion of an epicardial coronary artery 
• Subendocardial infarcts 
• Exclusively involve innermost layers of the myocardium (usually of the 
left ventricle, ventricular septum, or papillary muscles) 
• Subendocardium is MORE susceptible to ischemia because: 
• It is the zone subjected to highest pressure from the ventricular chamber 
• It has few collateral connections that supply it 
• It is perfused by vessels that must pass through layers of contracting 
myocardium
Factors Determining Infarct Size 
• The amount of tissue that infarcts relates to: 
• The mass of myocardium perfused by the occluded vessel 
• The magnitude and duration of impaired coronary blood flow 
• The oxygen demand of the affected region 
• The adequacy of collateral vessels that provide blood flow 
• The degree of tissue response that modifies the ischemic process
Arrhythmias 
• Occur frequently during acute MI and are a major source of mortality 
prior to hospital arrival 
• Upon arrival, arrhythmia associated deaths are uncommon 
• Due to anatomic interruption of blood flow to conduction structures 
• Accumulation of toxic metabolic products 
• Membrane leaks causing abnormal cellular ion concentrations 
• Autonomic stimulation 
• Administration of arrhythmogenic drugs 
• Detection: EKG
Ventricular arrhythmias within 48 hours 
solely suggest unstable electrical currents, 
but after 48 hours it is an indication for 
ICD implantation
Pericarditis 
• Early in post-MI period: Days 1-3 
• Cause: inflammation from necrosis/healing spreads from myocardium 
to pericardium 
• Detection: 
• Sharp pleuritic pain 
• Fever 
• Pericardial Friction Rub 
• Resolved with aspirin 
• Incidence limited by acute reperfusion strategies post MI 
• Anticoagulants contraindicated
Mechanical Complications 
• LV Papillary Muscle Rupture: 
• Detection: Loud holosystolic murmur due to severe acute mitral regurgitation  may be fatal 
• Partial rupture  moderate regurgitation  sxs of heart failure or pulmonary edema 
• Posteromedial LV papillary muscle is more susceptible 
• Ventricular Free Wall Rupture (rare/infrequent): occurs within 2 weeks following 
MI 
• Necrotic myocardium  free wall rupture hemorrhage into pericardial space  rapid cardiac 
tamponade (restricts ventricular filling) 
• Fatal 
• Detection: Imaging studies  surgical repair 
• Ventricular Septal Rupture 3-7 days 
• Blood shunted from LV to RV  congestive heart failure (JVD) 
• Detection: Loud holosystolic murmur @ Left Sternal Border (transseptal flow) 
• Use Doppler echocardiography to distinguish between acute mitral regurgitation (or O2 
saturation in chambers) 
• True Ventricular Aneurysm: Late complication (weeks to months after MI) 
• Weakened ventricular wall due to phagocytic clearance of necrotic tissue outward bulge 
• Suspected with persistent ST segment elevations on ECG (weeks later) and/or bulge on CXR 
• Detection: Confirmed by echocardiography
Holosystolic Murmurs 
• VSD 
• Mitral Regurgitation 
• Tricuspid Regurgitation
Heart Failure 
• Due to impaired LV contractility (systolic dysfunction) and myocardial 
stiffness (diastolic dysfunction) 
• Detection: 
• Dyspnea 
• Pulmonary Rales 
• Third heart sound (S3) 
• Rx: ACE inhibitors, Diuretics, Beta blockers, 
• Beta Agonists 
• Vasodilators
Cardiogenic Shock 
• When >40% of LV mass has infarcted 
• Decreased Cardiac Output (CO) 
• Hypotension: Systolic BP <90 mmHg 
• Low BP  decreased coronary perfusion  increases ischemic 
damage  decreased stroke volume increased LV size  
enhances myocardial oxygen demand 
• Mortality >70%
PRESSURE VOLUME 
LOOPS
Diagram a pressure-volume loop
ESPVR and EDPVR 
• The end-diastolic pressure-volume relationship is the passive filling 
curve of the ventricle, the slope of which is the inverse of ventricular 
compliance. (A completely stiff, non-compliant ventricle would have a 
really steep slope.) 
• The end-systolic pressure-volume relationship is the max pressure 
that the ventricle can muster for a specific volume given a specific 
inotropic state. The PV curves cannot “cross” this limit.
Preload 
• Preload is the initial stretching of cardiac myocytes prior to 
contraction. EDV and EDP are used as estimates. 
• Increasing venous return and ventricular preload leads to an increase in SV.
Determinants 
• Factors directly proportional to preload 
• Venous pressure - Increase in venous BP → more atrial filling → more ventricular filling 
• Ventricular compliance - increased compliance, increased ventricular filling at given pressure 
• Atrial inotropy - increase in atrial contraction by sympathetic activation can enhance 
ventricular filling 
• Outflow resistance -Increase in outflow resistance impairs the ability of the RV to empty  
Increase in preload. 
• Examples: Pulmonic valve stenosis, pulmonary hypertension, aortic valve stenosis, elevated aortic 
pressure 
• Factors inversely proportional to preload 
• Heart rate - Increased heart rate → Less time in diastole → less time for filling → lower 
preload. 
• Inflow resistance - Elevated inflow resistance reduces the rate of ventricular filling and 
decreases ventricular preload 
• Example: Tricuspid valve stenosis, mitral valve stenosis 
• Ventricular inotropy - Reduced inotropy → higher end-systolic volume → blood “backs up” in 
ventricle and proximal venous circulation → increased preload 
• Venous compliance - i.e. venodilation by NO increases compliance, reducing preload and O2 
demand.
Ventricular Compliance 
• Compliance = ΔV/ΔP, but usually we think of it as the inverse of the 
slope of passive filling on a PV curve. 
• Ventricular compliance is determined by the physical properties of the 
tissues making up the ventricular wall and the state of ventricular 
relaxation.
Length Tension Relationship 
• Length ∝ tension. When a myocyte is stretched, it’s passive tension 
increases (like a stretched rubber band) and it’s active tension 
increases (more forceful contraction when electrically stimulated). 
This means that increasing preload aka End Diastolic Pressure will 
create a bigger End Systolic Pressure (afterload).
Describe the three possible explanations 
for length-dependent activation. 
• Increased sarcomere length sensitizes troponin C to calcium. 
• Fiber stretching alters calcium homeostasis 
• Actin and myosin are brought in closer proximity to each other
Frank Starling Mechanism
Afterload 
• Afterload- The load against which the heart must contract to eject 
blood. 
• Left ventricle afterload ~ aortic pressure 
• Right ventricle afterload ~ pulmonary artery pressure
Wall Stress 
• La Place Equation: Wall stress, σ, is the average tension each 
myocyte must produce to shorten against the intraventricular 
pressure. Wall stress is directly proportional to afterload. 
• P = intraventricular pressure, r = ventricular radius, h = wall thickness 
• Intraventricular pressure is directly proportional to wall stress 
• Ventricular chamber dilatation increases wall stress (increased R) 
• Hypertrophy decreases wall stress (increased H)
Afterload and Velocity of Contraction 
• Afterload is like the weight that the muscle must lift, so increasing 
afterload (force) decreases velocity of contraction.
Preload and Velocity of Contraction 
• Increasing preload increases velocity of contraction for a given 
afterload. However, note that the y-intercepts are equal, meaning that 
the theoretical max velocity of contraction (what would occur against 
zero force) remains the same.
Afterload and Stroke Volume 
• At a given preload, increasing afterload decreases stroke volume.
What is a normal ejection fraction 
•55%
With changes in preload, EDV will change 
and with changes in afterload, ESV will 
change.
Inotropy 
• Increased Inotropy  Increased Stroke Volume
Inotropy and ESVPR 
• Increased inotropy shifts ESPVR (end systolic pressure volume 
relationship) to the left and makes it steeper because the ventricle 
can generate increased pressure at any given volume 
• Increasing Inotropy also increases stroke volume and ejection fraction 
(EF)
Determinants of Inotropy 
• Sympathetic nerve activation: Sympathetic nerves release 
norepinephrine which binds to β-1 adrenoceptors on myocytes and 
play a role in ventricular and atrial inotropic regulation 
• Circulating catecholamines: have positive inotropic effects. 
• Afterload: an increase in afterload can cause modest increase in 
inotropy by a somewhat unknown mechanism. 
• Heart rate: increased heart rate has positive inotropic effects. 
• This is due to an inability of the Na+/K+ ATPase to keep up with the sodium influx at 
higher frequency of action potential at elevated heart rates leading to an 
accumulation of intracellular calcium via sodium calcium exchanger.
Preload, Afterload and Inotropy
Dynamic effects of Increased Preload 
• Primary change: Increased EDV and SV (right shift, solid red line) 
• Secondary change: Increased afterload (due to increased CO and 
BP). 
• Inotropy is not affected. ESV also increases slightly due to higher 
afterload.
Cardiovascular Review
Cardiovascular Review
Cardiovascular Review
Cardiovascular Review
Cardiovascular Review
Cardiovascular Review
Cardiovascular Review
Cardiovascular Review
Cardiovascular Review
Cardiovascular Review
Cardiovascular Review
Cardiovascular Review
Cardiovascular Review
Cardiovascular Review
Cardiovascular Review
Cardiovascular Review
Cardiovascular Review
Cardiovascular Review
Cardiovascular Review
Cardiovascular Review
Cardiovascular Review
Cardiovascular Review
Cardiovascular Review
Cardiovascular Review
Cardiovascular Review
Cardiovascular Review
Cardiovascular Review
Cardiovascular Review
Cardiovascular Review
Cardiovascular Review
Cardiovascular Review
Cardiovascular Review
Cardiovascular Review
Cardiovascular Review
Cardiovascular Review
Cardiovascular Review
Cardiovascular Review
Cardiovascular Review
Cardiovascular Review
Cardiovascular Review
Cardiovascular Review
Cardiovascular Review
Cardiovascular Review
Cardiovascular Review
Cardiovascular Review
Cardiovascular Review
Cardiovascular Review
Cardiovascular Review
Cardiovascular Review
Cardiovascular Review
Cardiovascular Review
Cardiovascular Review
Cardiovascular Review
Cardiovascular Review
Cardiovascular Review
Cardiovascular Review
Cardiovascular Review
Cardiovascular Review
Cardiovascular Review
Cardiovascular Review
Cardiovascular Review
Cardiovascular Review
Cardiovascular Review
Cardiovascular Review
Cardiovascular Review
Cardiovascular Review
Cardiovascular Review
Cardiovascular Review
Cardiovascular Review
Cardiovascular Review
Cardiovascular Review
Cardiovascular Review
Cardiovascular Review
Cardiovascular Review
Cardiovascular Review
Cardiovascular Review
Cardiovascular Review
Cardiovascular Review
Cardiovascular Review
Cardiovascular Review
Cardiovascular Review
Cardiovascular Review
Cardiovascular Review
Cardiovascular Review
Cardiovascular Review
Cardiovascular Review
Cardiovascular Review
Cardiovascular Review
Cardiovascular Review
Cardiovascular Review
Cardiovascular Review
Cardiovascular Review
Cardiovascular Review
Cardiovascular Review
Cardiovascular Review
Cardiovascular Review
Cardiovascular Review
Cardiovascular Review
Cardiovascular Review
Cardiovascular Review
Cardiovascular Review
Cardiovascular Review
Cardiovascular Review
Cardiovascular Review
Cardiovascular Review
Cardiovascular Review
Cardiovascular Review
Cardiovascular Review
Cardiovascular Review
Cardiovascular Review
Cardiovascular Review
Cardiovascular Review
Cardiovascular Review
Cardiovascular Review
Cardiovascular Review
Cardiovascular Review
Cardiovascular Review
Cardiovascular Review
Cardiovascular Review
Cardiovascular Review
Cardiovascular Review
Cardiovascular Review
Cardiovascular Review
Cardiovascular Review
Cardiovascular Review
Cardiovascular Review
Cardiovascular Review
Cardiovascular Review
Cardiovascular Review
Cardiovascular Review
Cardiovascular Review
Cardiovascular Review
Cardiovascular Review
Cardiovascular Review
Cardiovascular Review
Cardiovascular Review
Cardiovascular Review
Cardiovascular Review
Cardiovascular Review
Cardiovascular Review
Cardiovascular Review
Cardiovascular Review
Cardiovascular Review
Cardiovascular Review
Cardiovascular Review
Cardiovascular Review
Cardiovascular Review
Cardiovascular Review
Cardiovascular Review
Cardiovascular Review
Cardiovascular Review
Cardiovascular Review
Cardiovascular Review
Cardiovascular Review
Cardiovascular Review
Cardiovascular Review
Cardiovascular Review
Cardiovascular Review
Cardiovascular Review
Cardiovascular Review
Cardiovascular Review
Cardiovascular Review
Cardiovascular Review
Cardiovascular Review
Cardiovascular Review
Cardiovascular Review
Cardiovascular Review
Cardiovascular Review
Cardiovascular Review
Cardiovascular Review
Cardiovascular Review
Cardiovascular Review
Cardiovascular Review
Cardiovascular Review
Cardiovascular Review
Cardiovascular Review
Cardiovascular Review
Cardiovascular Review
Cardiovascular Review
Cardiovascular Review
Cardiovascular Review
Cardiovascular Review
Cardiovascular Review
Cardiovascular Review
Cardiovascular Review
Cardiovascular Review
Cardiovascular Review
Cardiovascular Review
Cardiovascular Review
Cardiovascular Review
Cardiovascular Review
Cardiovascular Review
Cardiovascular Review
Cardiovascular Review
Cardiovascular Review
Cardiovascular Review
Cardiovascular Review
Cardiovascular Review
Cardiovascular Review
Cardiovascular Review
Cardiovascular Review
Cardiovascular Review
Cardiovascular Review
Cardiovascular Review
Cardiovascular Review
Cardiovascular Review
Cardiovascular Review
Cardiovascular Review
Cardiovascular Review
Cardiovascular Review
Cardiovascular Review
Cardiovascular Review
Cardiovascular Review
Cardiovascular Review
Cardiovascular Review
Cardiovascular Review
Cardiovascular Review
Cardiovascular Review
Cardiovascular Review
Cardiovascular Review
Cardiovascular Review
Cardiovascular Review
Cardiovascular Review
Cardiovascular Review
Cardiovascular Review
Cardiovascular Review
Cardiovascular Review
Cardiovascular Review
Cardiovascular Review
Cardiovascular Review
Cardiovascular Review
Cardiovascular Review
Cardiovascular Review
Cardiovascular Review
Cardiovascular Review
Cardiovascular Review
Cardiovascular Review
Cardiovascular Review
Cardiovascular Review
Cardiovascular Review
Cardiovascular Review
Cardiovascular Review
Cardiovascular Review
Cardiovascular Review
Cardiovascular Review
Cardiovascular Review
Cardiovascular Review
Cardiovascular Review
Cardiovascular Review
Cardiovascular Review
Cardiovascular Review
Cardiovascular Review
Cardiovascular Review
Cardiovascular Review
Cardiovascular Review
Cardiovascular Review
Cardiovascular Review
Cardiovascular Review
Cardiovascular Review
Cardiovascular Review
Cardiovascular Review
Cardiovascular Review
Cardiovascular Review
Cardiovascular Review
Cardiovascular Review
Cardiovascular Review
Cardiovascular Review
Cardiovascular Review
Cardiovascular Review
Cardiovascular Review
Cardiovascular Review
Cardiovascular Review
Cardiovascular Review
Cardiovascular Review
Cardiovascular Review
Cardiovascular Review
Cardiovascular Review
Cardiovascular Review
Cardiovascular Review
Cardiovascular Review
Cardiovascular Review
Cardiovascular Review
Cardiovascular Review
Cardiovascular Review
Cardiovascular Review
Cardiovascular Review
Cardiovascular Review
Cardiovascular Review
Cardiovascular Review
Cardiovascular Review
Cardiovascular Review
Cardiovascular Review
Cardiovascular Review
Cardiovascular Review
Cardiovascular Review
Cardiovascular Review
Cardiovascular Review
Cardiovascular Review
Cardiovascular Review
Cardiovascular Review
Cardiovascular Review
Cardiovascular Review
Cardiovascular Review
Cardiovascular Review
Cardiovascular Review
Cardiovascular Review
Cardiovascular Review
Cardiovascular Review
Cardiovascular Review
Cardiovascular Review
Cardiovascular Review
Cardiovascular Review
Cardiovascular Review
Cardiovascular Review
Cardiovascular Review
Cardiovascular Review
Cardiovascular Review

More Related Content

What's hot

What's hot (20)

Hyperlipidemia
HyperlipidemiaHyperlipidemia
Hyperlipidemia
 
Hyperlipidemia
HyperlipidemiaHyperlipidemia
Hyperlipidemia
 
Hyperlipidemia
HyperlipidemiaHyperlipidemia
Hyperlipidemia
 
Hyperlipidemias
HyperlipidemiasHyperlipidemias
Hyperlipidemias
 
Hyperlipidemia
Hyperlipidemia Hyperlipidemia
Hyperlipidemia
 
Hyperlipidemia
HyperlipidemiaHyperlipidemia
Hyperlipidemia
 
Hyperlipidemia
HyperlipidemiaHyperlipidemia
Hyperlipidemia
 
Pharmacology: Anti hyperlipidemic drugs flashcards
Pharmacology: Anti hyperlipidemic drugs flashcardsPharmacology: Anti hyperlipidemic drugs flashcards
Pharmacology: Anti hyperlipidemic drugs flashcards
 
Dyslipidemia
DyslipidemiaDyslipidemia
Dyslipidemia
 
Anti hyperlipidemic-drugs-presentation
Anti hyperlipidemic-drugs-presentationAnti hyperlipidemic-drugs-presentation
Anti hyperlipidemic-drugs-presentation
 
lipid lowering and atherosclerosis
lipid lowering and atherosclerosislipid lowering and atherosclerosis
lipid lowering and atherosclerosis
 
NurseReview.Org - Antilipemics Updates (medical pharmacology)
NurseReview.Org - Antilipemics Updates (medical pharmacology)NurseReview.Org - Antilipemics Updates (medical pharmacology)
NurseReview.Org - Antilipemics Updates (medical pharmacology)
 
Hyperlipidemia, pharmacology
Hyperlipidemia, pharmacologyHyperlipidemia, pharmacology
Hyperlipidemia, pharmacology
 
Hyperlipidemia and its treatment
Hyperlipidemia and its treatment Hyperlipidemia and its treatment
Hyperlipidemia and its treatment
 
Statins (report biopharm) Pravastatin and Rosuvastatin
Statins (report biopharm) Pravastatin and RosuvastatinStatins (report biopharm) Pravastatin and Rosuvastatin
Statins (report biopharm) Pravastatin and Rosuvastatin
 
Drugs used in Hyperlipidemia
Drugs used in HyperlipidemiaDrugs used in Hyperlipidemia
Drugs used in Hyperlipidemia
 
7 antihyperlipidemic
7 antihyperlipidemic7 antihyperlipidemic
7 antihyperlipidemic
 
Class hypolipidemics
Class hypolipidemics Class hypolipidemics
Class hypolipidemics
 
Hyperlipidemia
HyperlipidemiaHyperlipidemia
Hyperlipidemia
 
Drugs for Dyslipidemia
Drugs for DyslipidemiaDrugs for Dyslipidemia
Drugs for Dyslipidemia
 

Similar to Cardiovascular Review

Hyperlipidemia by Alveena urooj.pptx
Hyperlipidemia by Alveena urooj.pptxHyperlipidemia by Alveena urooj.pptx
Hyperlipidemia by Alveena urooj.pptx
spongybob1
 
Ped488 dyslipidemia s 11
Ped488 dyslipidemia s 11Ped488 dyslipidemia s 11
Ped488 dyslipidemia s 11
mjpol
 

Similar to Cardiovascular Review (20)

14 Dyslipidemia.pptx
14 Dyslipidemia.pptx14 Dyslipidemia.pptx
14 Dyslipidemia.pptx
 
Dyslipidaemia presentation
Dyslipidaemia presentationDyslipidaemia presentation
Dyslipidaemia presentation
 
Hyperlipidemia.pptx
Hyperlipidemia.pptxHyperlipidemia.pptx
Hyperlipidemia.pptx
 
Antihiperlipidemia varga 2021
Antihiperlipidemia varga 2021Antihiperlipidemia varga 2021
Antihiperlipidemia varga 2021
 
Drugs for Hyperlipoproteinemia.ppt
Drugs for Hyperlipoproteinemia.pptDrugs for Hyperlipoproteinemia.ppt
Drugs for Hyperlipoproteinemia.ppt
 
Dyslipidemia in a high risk patient
Dyslipidemia in a high risk patientDyslipidemia in a high risk patient
Dyslipidemia in a high risk patient
 
Antihyperlipidimic drug therapy-current and noval approaches.pptx
Antihyperlipidimic drug therapy-current and noval approaches.pptxAntihyperlipidimic drug therapy-current and noval approaches.pptx
Antihyperlipidimic drug therapy-current and noval approaches.pptx
 
Dyslipidemia approach
Dyslipidemia approachDyslipidemia approach
Dyslipidemia approach
 
Agents Used in Dyslipidemia.pptx
Agents Used in Dyslipidemia.pptxAgents Used in Dyslipidemia.pptx
Agents Used in Dyslipidemia.pptx
 
Lipid metabolism and hypolipedemic drugs
Lipid metabolism and hypolipedemic drugsLipid metabolism and hypolipedemic drugs
Lipid metabolism and hypolipedemic drugs
 
Hypolipidemic drugs.ppt
Hypolipidemic drugs.pptHypolipidemic drugs.ppt
Hypolipidemic drugs.ppt
 
Diabetic dyslipidemia
Diabetic dyslipidemiaDiabetic dyslipidemia
Diabetic dyslipidemia
 
Management of dyslipidemia
Management of dyslipidemiaManagement of dyslipidemia
Management of dyslipidemia
 
Hypolipidaemic Drugs
Hypolipidaemic DrugsHypolipidaemic Drugs
Hypolipidaemic Drugs
 
Hyperlipidemia by Alveena urooj.pptx
Hyperlipidemia by Alveena urooj.pptxHyperlipidemia by Alveena urooj.pptx
Hyperlipidemia by Alveena urooj.pptx
 
Ped488 dyslipidemia s 11
Ped488 dyslipidemia s 11Ped488 dyslipidemia s 11
Ped488 dyslipidemia s 11
 
Hypolipidemic drugs
Hypolipidemic drugsHypolipidemic drugs
Hypolipidemic drugs
 
Statins
StatinsStatins
Statins
 
Complications of abnormal lipid levels
Complications of abnormal lipid levelsComplications of abnormal lipid levels
Complications of abnormal lipid levels
 
Dyslipidemia2019
Dyslipidemia2019Dyslipidemia2019
Dyslipidemia2019
 

More from Jess Little (17)

Renal Histology
Renal HistologyRenal Histology
Renal Histology
 
Renal Review
Renal ReviewRenal Review
Renal Review
 
Pulmonology Radiology
Pulmonology RadiologyPulmonology Radiology
Pulmonology Radiology
 
Pulmonology Histology
Pulmonology HistologyPulmonology Histology
Pulmonology Histology
 
Pulmonology Review
Pulmonology ReviewPulmonology Review
Pulmonology Review
 
Mind Brain and Behavior
Mind Brain and BehaviorMind Brain and Behavior
Mind Brain and Behavior
 
MBB Review
MBB ReviewMBB Review
MBB Review
 
MBB Localizing Lesions
MBB Localizing LesionsMBB Localizing Lesions
MBB Localizing Lesions
 
MBB Anatomy
MBB AnatomyMBB Anatomy
MBB Anatomy
 
Mbb 2 b
Mbb 2 bMbb 2 b
Mbb 2 b
 
GI anatomy review
GI anatomy reviewGI anatomy review
GI anatomy review
 
GI Review
GI ReviewGI Review
GI Review
 
Cardiovascular Histology
Cardiovascular HistologyCardiovascular Histology
Cardiovascular Histology
 
Cardiovascular Drugs
Cardiovascular DrugsCardiovascular Drugs
Cardiovascular Drugs
 
Cardiovascular Anatomy
Cardiovascular AnatomyCardiovascular Anatomy
Cardiovascular Anatomy
 
Endorepro review
Endorepro reviewEndorepro review
Endorepro review
 
Endo Repro Anatomy and Histology
Endo Repro Anatomy and HistologyEndo Repro Anatomy and Histology
Endo Repro Anatomy and Histology
 

Recently uploaded

Call Girls in Lucknow Just Call 👉👉7877925207 Top Class Call Girl Service Avai...
Call Girls in Lucknow Just Call 👉👉7877925207 Top Class Call Girl Service Avai...Call Girls in Lucknow Just Call 👉👉7877925207 Top Class Call Girl Service Avai...
Call Girls in Lucknow Just Call 👉👉7877925207 Top Class Call Girl Service Avai...
adilkhan87451
 
🌹Attapur⬅️ Vip Call Girls Hyderabad 📱9352852248 Book Well Trand Call Girls In...
🌹Attapur⬅️ Vip Call Girls Hyderabad 📱9352852248 Book Well Trand Call Girls In...🌹Attapur⬅️ Vip Call Girls Hyderabad 📱9352852248 Book Well Trand Call Girls In...
🌹Attapur⬅️ Vip Call Girls Hyderabad 📱9352852248 Book Well Trand Call Girls In...
Call Girls In Delhi Whatsup 9873940964 Enjoy Unlimited Pleasure
 
💚Call Girls In Amritsar 💯Anvi 📲🔝8725944379🔝Amritsar Call Girl No💰Advance Cash...
💚Call Girls In Amritsar 💯Anvi 📲🔝8725944379🔝Amritsar Call Girl No💰Advance Cash...💚Call Girls In Amritsar 💯Anvi 📲🔝8725944379🔝Amritsar Call Girl No💰Advance Cash...
💚Call Girls In Amritsar 💯Anvi 📲🔝8725944379🔝Amritsar Call Girl No💰Advance Cash...
Sheetaleventcompany
 
Call Girl In Pune 👉 Just CALL ME: 9352988975 💋 Call Out Call Both With High p...
Call Girl In Pune 👉 Just CALL ME: 9352988975 💋 Call Out Call Both With High p...Call Girl In Pune 👉 Just CALL ME: 9352988975 💋 Call Out Call Both With High p...
Call Girl In Pune 👉 Just CALL ME: 9352988975 💋 Call Out Call Both With High p...
chetankumar9855
 

Recently uploaded (20)

Call Girls Service Jaipur {9521753030 } ❤️VVIP BHAWNA Call Girl in Jaipur Raj...
Call Girls Service Jaipur {9521753030 } ❤️VVIP BHAWNA Call Girl in Jaipur Raj...Call Girls Service Jaipur {9521753030 } ❤️VVIP BHAWNA Call Girl in Jaipur Raj...
Call Girls Service Jaipur {9521753030 } ❤️VVIP BHAWNA Call Girl in Jaipur Raj...
 
Kollam call girls Mallu aunty service 7877702510
Kollam call girls Mallu aunty service 7877702510Kollam call girls Mallu aunty service 7877702510
Kollam call girls Mallu aunty service 7877702510
 
Coimbatore Call Girls in Thudiyalur : 7427069034 High Profile Model Escorts |...
Coimbatore Call Girls in Thudiyalur : 7427069034 High Profile Model Escorts |...Coimbatore Call Girls in Thudiyalur : 7427069034 High Profile Model Escorts |...
Coimbatore Call Girls in Thudiyalur : 7427069034 High Profile Model Escorts |...
 
Call Girls Service Jaipur {8445551418} ❤️VVIP BHAWNA Call Girl in Jaipur Raja...
Call Girls Service Jaipur {8445551418} ❤️VVIP BHAWNA Call Girl in Jaipur Raja...Call Girls Service Jaipur {8445551418} ❤️VVIP BHAWNA Call Girl in Jaipur Raja...
Call Girls Service Jaipur {8445551418} ❤️VVIP BHAWNA Call Girl in Jaipur Raja...
 
Call Girls in Lucknow Just Call 👉👉7877925207 Top Class Call Girl Service Avai...
Call Girls in Lucknow Just Call 👉👉7877925207 Top Class Call Girl Service Avai...Call Girls in Lucknow Just Call 👉👉7877925207 Top Class Call Girl Service Avai...
Call Girls in Lucknow Just Call 👉👉7877925207 Top Class Call Girl Service Avai...
 
Call Girls Mysore Just Call 8250077686 Top Class Call Girl Service Available
Call Girls Mysore Just Call 8250077686 Top Class Call Girl Service AvailableCall Girls Mysore Just Call 8250077686 Top Class Call Girl Service Available
Call Girls Mysore Just Call 8250077686 Top Class Call Girl Service Available
 
Call Girls Hyderabad Just Call 8250077686 Top Class Call Girl Service Available
Call Girls Hyderabad Just Call 8250077686 Top Class Call Girl Service AvailableCall Girls Hyderabad Just Call 8250077686 Top Class Call Girl Service Available
Call Girls Hyderabad Just Call 8250077686 Top Class Call Girl Service Available
 
Premium Bangalore Call Girls Jigani Dail 6378878445 Escort Service For Hot Ma...
Premium Bangalore Call Girls Jigani Dail 6378878445 Escort Service For Hot Ma...Premium Bangalore Call Girls Jigani Dail 6378878445 Escort Service For Hot Ma...
Premium Bangalore Call Girls Jigani Dail 6378878445 Escort Service For Hot Ma...
 
Independent Call Girls Service Mohali Sector 116 | 6367187148 | Call Girl Ser...
Independent Call Girls Service Mohali Sector 116 | 6367187148 | Call Girl Ser...Independent Call Girls Service Mohali Sector 116 | 6367187148 | Call Girl Ser...
Independent Call Girls Service Mohali Sector 116 | 6367187148 | Call Girl Ser...
 
Call Girls Jaipur Just Call 9521753030 Top Class Call Girl Service Available
Call Girls Jaipur Just Call 9521753030 Top Class Call Girl Service AvailableCall Girls Jaipur Just Call 9521753030 Top Class Call Girl Service Available
Call Girls Jaipur Just Call 9521753030 Top Class Call Girl Service Available
 
Call Girls Mumbai Just Call 8250077686 Top Class Call Girl Service Available
Call Girls Mumbai Just Call 8250077686 Top Class Call Girl Service AvailableCall Girls Mumbai Just Call 8250077686 Top Class Call Girl Service Available
Call Girls Mumbai Just Call 8250077686 Top Class Call Girl Service Available
 
Top Quality Call Girl Service Kalyanpur 6378878445 Available Call Girls Any Time
Top Quality Call Girl Service Kalyanpur 6378878445 Available Call Girls Any TimeTop Quality Call Girl Service Kalyanpur 6378878445 Available Call Girls Any Time
Top Quality Call Girl Service Kalyanpur 6378878445 Available Call Girls Any Time
 
Call Girls Kolkata Kalikapur 💯Call Us 🔝 8005736733 🔝 💃 Top Class Call Girl Se...
Call Girls Kolkata Kalikapur 💯Call Us 🔝 8005736733 🔝 💃 Top Class Call Girl Se...Call Girls Kolkata Kalikapur 💯Call Us 🔝 8005736733 🔝 💃 Top Class Call Girl Se...
Call Girls Kolkata Kalikapur 💯Call Us 🔝 8005736733 🔝 💃 Top Class Call Girl Se...
 
Top Rated Hyderabad Call Girls Chintal ⟟ 9332606886 ⟟ Call Me For Genuine Se...
Top Rated  Hyderabad Call Girls Chintal ⟟ 9332606886 ⟟ Call Me For Genuine Se...Top Rated  Hyderabad Call Girls Chintal ⟟ 9332606886 ⟟ Call Me For Genuine Se...
Top Rated Hyderabad Call Girls Chintal ⟟ 9332606886 ⟟ Call Me For Genuine Se...
 
Call Girls Rishikesh Just Call 8250077686 Top Class Call Girl Service Available
Call Girls Rishikesh Just Call 8250077686 Top Class Call Girl Service AvailableCall Girls Rishikesh Just Call 8250077686 Top Class Call Girl Service Available
Call Girls Rishikesh Just Call 8250077686 Top Class Call Girl Service Available
 
🌹Attapur⬅️ Vip Call Girls Hyderabad 📱9352852248 Book Well Trand Call Girls In...
🌹Attapur⬅️ Vip Call Girls Hyderabad 📱9352852248 Book Well Trand Call Girls In...🌹Attapur⬅️ Vip Call Girls Hyderabad 📱9352852248 Book Well Trand Call Girls In...
🌹Attapur⬅️ Vip Call Girls Hyderabad 📱9352852248 Book Well Trand Call Girls In...
 
Andheri East ) Call Girls in Mumbai Phone No 9004268417 Elite Escort Service ...
Andheri East ) Call Girls in Mumbai Phone No 9004268417 Elite Escort Service ...Andheri East ) Call Girls in Mumbai Phone No 9004268417 Elite Escort Service ...
Andheri East ) Call Girls in Mumbai Phone No 9004268417 Elite Escort Service ...
 
💚Call Girls In Amritsar 💯Anvi 📲🔝8725944379🔝Amritsar Call Girl No💰Advance Cash...
💚Call Girls In Amritsar 💯Anvi 📲🔝8725944379🔝Amritsar Call Girl No💰Advance Cash...💚Call Girls In Amritsar 💯Anvi 📲🔝8725944379🔝Amritsar Call Girl No💰Advance Cash...
💚Call Girls In Amritsar 💯Anvi 📲🔝8725944379🔝Amritsar Call Girl No💰Advance Cash...
 
Call Girl In Pune 👉 Just CALL ME: 9352988975 💋 Call Out Call Both With High p...
Call Girl In Pune 👉 Just CALL ME: 9352988975 💋 Call Out Call Both With High p...Call Girl In Pune 👉 Just CALL ME: 9352988975 💋 Call Out Call Both With High p...
Call Girl In Pune 👉 Just CALL ME: 9352988975 💋 Call Out Call Both With High p...
 
VIP Hyderabad Call Girls Bahadurpally 7877925207 ₹5000 To 25K With AC Room 💚😋
VIP Hyderabad Call Girls Bahadurpally 7877925207 ₹5000 To 25K With AC Room 💚😋VIP Hyderabad Call Girls Bahadurpally 7877925207 ₹5000 To 25K With AC Room 💚😋
VIP Hyderabad Call Girls Bahadurpally 7877925207 ₹5000 To 25K With AC Room 💚😋
 

Cardiovascular Review

  • 3. Atherosclerosis is the leading cause of Morbidity and Mortality in Western Society Most Common Factors: Hypertension Smoking Hypercholesterolemia Diabetes
  • 4. Risk Factors • Dyslipidemia: When in excess, LDL accumulates in subendothelial space and can start to damage the intima, initiating and perpetuating development of atherosclerotic lesions. Treatments slow progression of atherosclerotic plaques. • Smoking: Tobacco smoking could lead to atherosclerotic disease in several ways, including enhanced oxidative modification of LDL, decreased circulating HDL levels, endothelial dysfunction owing to tissue hypoxia and increased oxidant stress, increased platelet adhesiveness among others. • Hypertension: Hypertension injures vascular endothelium and may increase the permeability of the vessel wall to lipoproteins. Antihypertensive results like lipid-reducing treatments, halting of atherosclerotic progression • DM: High blood sugar may result in nonenzymatic glycosylation of lipoproteins (which enhances uptake of cholesterol by scavenger macrophages, as described earlier), or to a prothrombotic tendency and antifibrinolytic state that is often present. Diabetics frequently have impaired endothelial function, gauged by the reduced bioavailability of NO and increased leukocyte adhesion. Managing DM may halt progression • Elevated CRP: Atherosclerosis is thought to arise from an inflammatory reaction; CRP is a marker of inflammation.
  • 6. Atherosclerosis is an INTIMAL PROCESS
  • 8. What are the steps of Atherosclerosis? • Endothelial Activation • LDL, monocytes, endothelium • Foam Cells • Oxidized LDL, macrophages • Fibrous Cap • SMCs • Calcification • Ulceration • Thrombosis
  • 9.
  • 10.
  • 11. Name the important molecules involved in mediating atherosclerosis • 1. Monocyte attaches to VCAM-1 • 2. MCP-1 is released to attract more monocytes • 3. Macrophages release MPO to oxidize LDL • 4. Scavenger receptor on macrophage allows LDL uptake • 5. Lipid core releases PDGF to stimulate SMCs • 6. SMCs release MMP, plaque is destabilized
  • 12. What are the important complications of atherosclerosis? • Occlusion of vessel • Thrombus • Ulceration and hemorrhage • Atheroembolism • Narrowing of lumen (stenosis) • Weakening of wall (aneurysm)
  • 15. Dangers of Complicated Plaques • Simple atheromas have a fibrous cap that conveys a great deal of plaque stability. Thus with simple atheromas, patients may present with arterial stenosis resulting in hypoperfusion but are unlikely to present with thrombotic or embolic events. • Complicated atheromas can have an ulcerated cap, which expose procoagulants in plaque to circulation increasing chance of thrombus at the site or lead to greater occlusion of vessel. Rupture of a plaque in complicated atheromas can also lead to hemorrhage. Also complicated atheromas can have calcifications, which can increase the fragility of the plaque and thus likelihood of thrombotic events.
  • 16. Atheroembolism will show cholesterol plaques
  • 18. Mechanism Competitively inhibit HMG-CoA reductase: (1) Decreases intracellular cholesterol  induces SREBP  increases expression of LDL-R (2) VLDL and IDL are cleared more rapidly due to cross-recognition with hepatic LDL-R (3) Hepatic VLDL production falls due to reduced cholesterol availability  reduced LDL and triglycerides Modify platelets and endothelium (e.g., enhanced NO synthesis) Suppress inflammation Effects Decreases LDL 18-55% Decreases TG 7-30% Increases HDL 5-15% (unclear mechanism) Side Effects Myopathy (increased w/ niacin, fibrates), hepatotoxicity, drug interactions (CYP3A4 inhibition: macrolides, azoles, HIV protease inhibitors) Statins
  • 19. Mechanism Inhibits NPC1L1 at brush border of epithelial cells in small intestine  reduced chylomicron production  less cholesterol delivered to liver  compensatory increase in hepatic LDL-R  increased LDL clearance Effects Decreases LDL 18% Has additive effect w/ statins and fibrates on LDL Side Effects Muscle weakness (slightly higher w/ statin), transaminitis (slightly higher w/ statin) Ezetimibe
  • 20. Bile Acid Resins Mechanism (+)-charged amines bind (-)-charged bile acids and prevent recycling in liver hepatic cholesterol (FXR, CYP7A ) LDL-R UNLIKE statins, new cholesterol production is stimulated (b/c HMG-CoA reductase is not inhibited)  VLDL production  serum TGs Effects Decreases LDL 15-30% Increases HDL 4% No change/slight increase in TGs Side Effects GI, drug interactions (fat-soluble drugs: esp warfarin, digoxin), absorption of fat-soluble vitamins, pancreatitis, cholesterol gallstones *Not absorbed systemically, so no systemic side effects*
  • 21. Niacin (Vitamin B3) Mechanism Decreases lipolysis in adipose tissue  less FAs available for TG synthesis in liver Decreases VLDL synthesis, so less LDL Increases HDL by decreasing hepatic removal of HDL Effects Decreases LDL 5-25% Increases HDL 15-30% Decreases TGs 20-50% Side Effects Cutaneous flushing (due to prostaglandins; take aspirin), GI (nausea, PUD), hepatotoxicity, insulin resistance and hyperglycemia (caution w/ diabetics), gout (raises serum uric acid levels), myopathy (increases w/ statin)
  • 22. Mechanism Activate PPARα-RXR  (1) Enhanced oxidation of FAs in liver and muscle  decreased TG levels  decreased VLDL (2) Increased expression of LPL (3) Increased rate of HDL-mediated reverse cholesterol transport (due to apo AI transcription) Effects Decreases LDL 5-20% Increases HDL 10-20% Decreases TGs 20-50% *Larger decreases in TGs and increases in HDL than statins. Side Effects GI (dyspepsia, abdominal pain, diarrhea), cholesterol gallstones, myopathy (increased w/ liver and kidney dysfunction; worse w/ statins), augment effects of oral hypoglycemic drugs (avoid in diabetes) Fibrates Gemfibrozil inhibits glucuronidation of most statins, which can increase statin-related side effects. Fenofibrate does not.
  • 23. Gemofibrizil vs. Fenofibrate • Drugs that block CYP3A4 slow statin metabolism, increasing the risk of side effects. • Gemfibrozil inhibits the glucuronidation of statins and increases the chances of side effects. • Choose Fenofibrate in patients taking Statins.
  • 24. Fish Oil (omega-3 FA) Mechanism Not well defined, ?agonist of PPARα, ?binds enzymes in TG synthesis but cannot be utilized Effects Variable effect on LDL Increases HDL 9% Decreases TGs 50% Side Effects Minimal, may prolong bleeding time (caution use w/ NSAIDs, ASA, warfarin), caution w/ shellfish hypersensitivity
  • 25. Major SE of Hyperlipidemia Tx
  • 26.
  • 27. 4 Major Statin Treatment Groups • Individuals with clinical atherosclerotic cardiovascular disease (ASCVD). • Tx recommendations: • High-intensity statin if < 75 years old • Moderate-intensity statin if > 75 years old (or < 75 y/o and not a candidate for high-intensity) • Individuals with primary elevations of LDL-C >190 mg/dL. • Tx recommendation: High-intensity statin (or moderate if not a candidate) • Individuals w/ diabetes aged 40-75 yrs with LDL-C 70-189 mg/dL and w/o clinical ASCVD. • Tx recommendation: • Estimated 10-year risk of ASCVD < 7.5%: Moderate-intensity statin • Estimated 10-year risk of ASCVD > 7.5%: High intensity statin • Individuals w/o clinical ASCVD or diabetes with LDL-C 70-189 mg/dL and 10 year ASCVD risk > 7.5%. • Tx recommendation: Moderate or high intensity statin
  • 28.
  • 29.
  • 30. HDL… Good or Bad? • HDL is an important player in Reverse Cholesterol Transport and removes cholesterol from circulation. Also there is an inverse correlation between HDL levels and CVD risk- high HDL = low CVD risk • Controversy: Two clinical trials aimed to increase HDL failed in phase III due to adverse off target effects and lack of efficacy. Another clinical trial using niacin that increases HDL showed no added benefit.
  • 31. Lifestyle Guidelines • Consume a dietary pattern that emphasizes intake of vegetables, fruits, and whole grains; includes low-fat dairy products, poultry, fish, legumes, nontropical vegetable oils, and nuts; and limits intake of sweets, sugar-sweetened beverages, and red meats. • Adapt this dietary pattern to appropriate calorie requirements, personal and cultural food preferences, and nutrition therapy for other medical conditions (including diabetes). • Achieve this pattern by following plans such as the DASH dietary pattern, the US Department of Agriculture (USDA) Food Pattern, or the AHA Diet. • Aim for a dietary pattern that achieves 5% to 6% of calories from saturated fat. • Reduce percentage of calories from saturated fat. • Reduce percentage of calories from trans fat. • Engage in aerobic physical activity to reduce LDL-C and non–HDL-C: 3 to 4 sessions per week, lasting on average 40 minutes per session, and involving moderate- to vigorous- intensity physical activity. •
  • 32. PCSK9 • PCSK9 is a protein that promotes the degradation of LDL receptors and decreases the amount of LDL receptors on cell surface. Any manipulation that will decrease the functionality or availability of PCSK9 will result in increased LDL receptors on cell surface and thus lower circulating LDL levels. • Potential Future RNA interference drug
  • 34. Define mean arterial pressure (MAP) and describe how changes in cardiac output (CO), systemic vascular resistance (SVR), and central venous pressure (CVP) affect MAP. • Mean arterial pressure (MAP) - the average arterial pressure during a single cardiac cycle. • MAP=(CO x SVR) + CVP • MAP = PDIAS + PPULSE/3 • MAP is not the arithmetic mean of PDIAS and PSYS since the heart spends twice as long in diastole than systole under resting conditions • Increases in the cardiac output (CO), systemic vascular resistance (SVR), and central venous pressure (SVR) will increase the mean arterial pressure.
  • 35. Compare the pressures, flows, and resistances in the pulmonary circulation with those in the systemic circulation. • Pressure: Pulmonary < Systemic • Flow: Pulmonary = Systemic • Resistance: Pulmonary < Systemic • Pulmonary v. Systemic- Pressures in the pulmonary circulation are not as high as the pressures found in the systemic circulation. The pressure in the pulmonary circulation is 25/10mmHg. The blood volume output has to be equal on both sides of the heart so as to ensure that there are no major blood volume changes between the pulmonary and systemic circulations. Although is there is resistance within the pulmonary circulation, it is less than the resistance found in system circulation.
  • 36. Define central venous pressure (CVP) and how this pressure relates to stroke volume and cardiac output (CO). • Central venous pressure (CVP) is the pressure found in the vena cava near the right atrium. • Same in same out. • CVP is a measure of how much blood is going into the heart, which is equal to how much blood is coming out of the heart. Blood flows from peripheral venous pool (PVP) to central venous pool (CVP). The difference in pressure between PVP and CVP is the driving force pushing blood into the heart. By increasing the difference in pressure between the two systems, you create a larger flow from PVP to CVP, which results in more blood going into the right atrium  GREATER STROKE VOLUME AND CARDIAC OUTPUT
  • 37. Explain how each of the following affects CVP: Blood volume, venous compliance, gravity, respiration, and muscle contraction. • Blood volume: increased blood volume increased venous return  increased CO • Venous compliance: decrease in venous compliance increases central venous pressure. • Gravity: Blood pools in LE  momentary decrease in venous return/CO before compensatory mechanisms kick in • Respiration: during inspiration, the intrapleural pressure decreases. This leads to a decrease in CVP, increase in pressure gradient, increase in venous return/CO • Muscle Contraction: muscle contraction in the leg helps facilitate the movement of blood from the veins in the lower extremities. This mechanism works against gravity to help increase CVP and increase CO.
  • 38. Define the formula that relates blood flow through an arteriole to (a) pressure difference down the length of an arteriole and (b) resistance to that flow. • The formula that relates blood flow (F) through an arteriole to pressure (P) down a length and resistance (R) is • Q = ΔP/R
  • 39. Explain what key factors are responsible for the pressure difference and resistance to flow. • Resistance depends upon viscosity of blood (n), vessel length (l) and vessel radius (r) • R = (n*l)/ (r^4) • Q = Delta P * (r ^4)/ (n*l) • Flow is directly related to pressure difference and inversely related to resistance
  • 40. Explain why the decrease in pressure across arterioles is much greater than the pressure drop across other vessel types. • Resistance is related to radius to the 4th power, vessel length and viscosity of the blood. • Resistance is strongly related to the diameter of the vessel lumen: the smaller the diameter of the vessel lumen, the greater the resistance. Taken as an individual vessel, the individual capillary would exhibit greater resistance than the individual arteriole, because the diameter of the typical capillary lumen is smaller, but, if one looks at the entire vascular system as a whole, much more of the resistance of the system is contributed by arterioles than by capillaries • There are so many more capillaries and with many capillaries in parallel, that allows collectively a lot of blood flow whereas arterioles tend to be found individually and they also tend to be much longer than capillaries (which also increases resistance). Thus, In the larger system, arterioles are the greater contributor to resistance.
  • 41. Define the formula that relates resistance to blood flow to (a) length of the arteriole, (b) viscosity of blood, and (c) internal radius of the arteriole. • Resistance = (viscosity * length)/ (radius^4) • The radius has the greatest effect (note it is to the fourth power), so vasoconstriction and vasodilation greatly affect resistance.
  • 42. Explain which one of these factors has the greatest influence on the resistance to blood flow and what in turn are the greatest influences on it. • Radius has greatest effect on flow because it is to the fourth power • Blood volume and vessel compliance, vasodilation/constriction, and pathologic sclerosis may affect radius.
  • 43. Predict the relative changes in flow through an arteriole caused by changes in arteriole length, arteriolar radius, fluid viscosity, and pressure difference. • Using the equations Q = ΔP/R and Resistance = (viscosity * length)/radius4 • Increase in length → more resistance → less flow • Increase in radius → less resistance → more flow • Increase in viscosity → more resistance → less flow • Increase in pressure difference → more flow
  • 44. Differentiate between the pressures and forces that influence the caliber of intra-alveolar capillaries and those that influence extra-alveolar resistance vessels. • Intra-alveolar capillaries are very closely associated with alveoli, which expand upon inhalation. This causes the capillaries to also stretch, thus decreasing their diameter. This results in a greater resistance. • For capillaries, increased lung volume means greater resistance • Extra-alveolar resistance vessels, on the other hand, are tethered to the pleural cavity. When the lung volume expands, the connective tissue attaching them to the pleural cavity pulls on their walls to expand their diameter • For bigger vessels, decreased lung volume means greater resistance • There is a sweet spot in the middle where the lung volume causes the resistance to be the least for both capillaries and the resistance vessels and this is the “working lung volume.”
  • 45. Explain the effects of gravity on pulmonary blood flow. • When standing up, CVP drops since venous blood pools in the legs. This decreases venous return and cardiac output, also decreasing pulmonary blood flow. Also when the person is standing up, the blood flow is lowest at the apex of the lung and highest at the base.
  • 46. Describe how pulmonary blood flow varies from the base of the lungs to the apex. • Due to effects of gravity, the base of the lung receives more blood than the apex. This leads to poor “V/Q” matching, since ventilation (V) is not matched to blood flow (Q) at different areas of the lung. • The V/Q ratio at the top of the lungs is high, while the V/Q ratio at the base of the lungs is low.
  • 47. Describe the changes in pulmonary vascular resistance when the pressures in pulmonary arteries and pulmonary veins increase. • Q = ΔP/R; • If pressure increases, resistance also increases proportionately in order to keep the flow constant. However, pulmonary circulation has a much lower resistance because the vessels have thinner walls, have less smooth muscle, are more distensible and they can do “recruitment”. Therefore, the pressure can remain low while maintaining the same flow
  • 48. Describe the roles of recruitment and distension in decreasing pulmonary vascular resistance. • The CO of the right and left sides of the heart are equal. Because the pressures on the systemic side is so much greater than on the pulmonary side, the pulmonary system must have much less resistance. Pulmonary circulation is able to accomplish this lower resistance with thinner vessel walls, less smooth muscle in the vessel walls, greater distensibility, and recruitment of non-perfused vessels. • Recruitment is adding new vessels in parallel circuit to lower the total resistance, while distension is decreasing the resistance in each vessel by expanding its diameter
  • 49. Use the Fick Principle to estimate pulmonary blood flow. • Assumption: in the steady state, the cardiac output of the L and R ventricles are equal • O2 consumption = Blood flow ( Arterial-venous O2 Difference)
  • 50. Explain the mechanism and significance of hypoxic vasoconstriction. • When PAO2 is normal (around 100 mmHg), O2 diffuses from the alveoli into the nearby arteriolar smooth muscle cells, keeping the arterioles relatively relaxed and dilated. When PAO2 decreases to below 70 mmHg, the smooth muscle cells can recognize the reduced amount of O2 coming in from the alveoli. This causes them to contract, reducing the amount of pulmonary blood flow to that region • Significance: • This reduces pulmonary blood flow to poorly ventilated areas. Pulmonary blood flow is directed away from poorly ventilated regions of the lung and redirected to other, better oxygenated regions
  • 52.
  • 53. PT- Extrinsic Pathway PTT – Intrinsic Pathway (TENET)
  • 54. Summarize the mechanism for the initiation of a blood clot and identify the essential enzymes involved in this mechanism. • Tissue Factor (TF) is usually in the subendothelial membrane on smooth muscle and collagen. When tissue is injured, it is released, activating the coagulation cascade. This activates the Extrinsic Pathway by forming a complex with factor 7a.
  • 55. Predict the effect of vitamin K deficiency on the coagulation system • Factors II, VII, IX, and X require gamma carboxylation from Vitamin K in order to bind Ca on the surface of a clot. Vitamin K is absorbed in the intestine and metabolized in the liver. Therefore, people with end-stage liver disease or people who are not adequately absorbing Vit K in their intestines will see prolongation of the PT. We see a prolongation of the PT because the Extrinsic Pathway is most affected. • Vitamin K is also required for activation of anticoagulant proteins C and S. These proteins are the first to drop in Vit K deficiency, so we often see a transient pro-thrombotic state before the clotting factors drop. • (This is especially important to remember with administration of heparin).
  • 56. Explain why a patient with end stage liver disease may have abnormal coagulation function. • Vitamin K is metabolized in the liver. Therefore, people with end stage liver disease are going to have a Vitamin K deficiency and won’t be able to activate Factors II, VII, IX, and X.
  • 57. Describe the conversion of fibrinogen to fibrin and the role of factor XIII in this reaction. • Thrombin (IIa) converts fibrinogen (I) into fibrin (Ia), which is necessary to form a clot. Once fibrinogen gets converted into fibrin, it creates fibrin bonds and forms a fibrin mesh with crosslinks. That mesh gets activated and covalently bonds in the presence of factor XIII (activated by thrombin). Factor XIII stabilizes the clot in normal hemostasis. Factor XIII does not prolong PT or PTT.
  • 58. Outline the steps of fibrinolysis and identify the inhibitors of fibrin degradation. • Tissue plasminogen activator (t-PA) is released from damaged vessels and cleaves plasminogen to the active enzyme plasmin. • In the circulation, plasminogen activator inhibitors 1 and 2 rapidly inactivate t-PA. • However, t-PA binds to fibrin locally at the site of release, and converts fibrin-bound plasminogen to plasmin. Plasmin splits both fibrinogen and fibrin into degradation products (D-dimer), and if this occurs at the site of a thrombus it produces lysis of the clot matrix.
  • 59. Diagram the formation of the D-dimer and explain its utility in diagnosis of venous thromboembolic disease. • D-dimer: cross-linked regions of fibrin that have been degraded - “you have a clot, you’ve broken it down, and now you have d-dimer in your blood” - specific for fibrinolysis (however, not fibrinogenolysis) and useful for excluding thrombosis
  • 60. Given values for various clotting factor concentrations, be able to predict which screening tests of coagulation will be abnormal. • PT (prothrombin time): tests function of common and extrinsic pathway (factors I (Fibrinogen), II (Prothrombin) , V, VII, and X). Defect in any of these factors => increased PT • PTT (partial thromboblastin time): tests function of common and intrinsic pathway (factors I, II, XII, XI, IX, VIII, X). Defect in any of these factors => increased PTT • Vitamin K deficiency => increase in both because factors II, VII, IX, X require Vitamin K, so all pathways are affected).
  • 61. Explain how activated protein C and antithrombin act as inhibitors of coagulation. • Thrombomodulin (TM), which is normally present on the endothelial cells, binds thrombin. • Protein C (a vitamin K dependent factor that is normally found in the blood) binds to the Thrombin/TM complex, and becomes activated as Activated Protein C. • Protein S binds activated Protein C (APC), accelerating its activity. • APC inhibits coagulation by inactivating Factors V and VIII.
  • 63. Define the roles of (a) If Na+-channels, (b) K+-channels, (c) voltage-gated Ca2+- channels, and (d) K-Ach channels, in pacemaker activity of SA cells. • a) If Na+-channels: Voltage and receptor gated. Contribute to phase 4 pacemaker funny current (repolarized state to depolarized state) due to a slow inward movement of Na+. • Note: “diastolic depolarization” refers to pacemaker phase/phase 4 and it represents the non-contracting time between heartbeats (diastole) • b) K+ channels: outflux of K+ in phase 3 contributes to repolarization • c) Voltage-gated Ca2+ channels: influx of Ca2+ during phase 0 contributes to depolarization • d) K-ACh channels: ACh activated K+ channels are activated by ACh and adenosine and are G-protein coupled. They slow SA nodal firing upon VAGAL stimulus. • 1. ACh is released from vagal nerve terminals (parasympathetic innervation of heart) near SA node • 2. ACh binds to ACh activated K+ channels and K+ channels open • 3. Resting potential approaches reversal potential for K+, about -90 mV • 4. As a result, SA cells take longer to reach threshold for firing
  • 64. Describe the effects and mechanisms by which NE and Ach affect the activity of these ions channels and therefore HR. • NE: causes positive chronotropy (increase in heart rate). • Sympathetic activation of SA node increases slope of phase 4 and lowers threshold, increasing pacemaker frequency • NE released by sympathetic adrenergic nerves binds to 훃1 adrenoreceptors (훃 1AR above) coupled to a stimulatory Gs-protein which activates adenylyl cyclase and increases cAMP • This leads to an increase in funny current (If) and an earlier opening of Ca2+ channels, both of which increase rate of depolarization • Ach: causes negative chronotropy (slowing of heart rate). • Vagal nerve stimulation releases ACh at SA node which decreases slope of phase 4 by inhibiting funny currents, hyperpolarizing the cell, and increasing threshold voltage required to trigger phase 0. • ACh binds to M2 receptors and decreases cAMP via inhibitory Gi protein • ACh also activates K-ACh channels that hyperpolarizes the cell by increasing K+ conductance
  • 65. Define the roles of (a) fast Na+-channels, (b) K+-channels, (c) voltage-gated Ca2+- channels, and (d) Na+-Ca2+ exchanger, in cardiac muscle contraction. • a) Fast Na+ channels: voltage-gated, open during phase 0, results in rapid depolarization. The upstrokes of all the ventricular muscle cell APs corresponds to the QRS complex • b) K+ channels: Two types; Transient outward type involved in phase 1 initial repolarization. Delayed rectifier type involved in phase 3 repolarization • c) Voltage-gated Ca2+ channels: contribute to slow inward, long-lasting current, plateau phase 2 • d) Na+-Ca2+ exchanger: one mechanism to remove calcium from cells after it accumulates after action potentials. 3 Na+ ions in exchanged for 1 Ca2+ ion out.
  • 66. Membrane Potentials • Chemical or Concentration Gradient: • created by concentration differences of ions • Electrostatic gradient: • positive ions will move to negative charge and vice-versa • Nernst Equation provides information about the membrane potential that is necessary to oppose the outward movement of a particular ion down its concentration gradient (the equilibrium potential of particular ion). • Ek=-61log[K+]inside/[K+]outside=-96 mV • But the membrane potential is also dependent on the conductance. This is where the semi-permeable characteristic of plasma membranes becomes important. • Em=g'K(EK)+g'Ca(ECa)+g'Na(ENa) • The equilibrium potential changes very little due to the relatively small changes in ion concentration. Thus the most important factor determining membrane potential is the differences in ion conductances.
  • 67. List the major ions involved in establishing the cardiac membrane potential. • Na+, K+, and Ca2+ are the major ions that dictate membrane potential.
  • 68. Define equilibrium potential and know its normal value for K+ and Na+ ions. • Equilibrium potential: the potential difference across the membrane required to maintain the concentration gradient across the membrane. • NOTE: Resting membrane potential for a cardiomyocyte is -90 mV (quite close to the equilibrium potential for K+). This is due to the fact that the membrane is much more permeable to K+ in a resting state.
  • 69. Sketch a typical action potential in a pacemaker cell.
  • 70. Sketch a typical action potential in a ventricular cell.
  • 71. Describe the ion channels that contribute to each phase of the cardiac AP. • Nodal: Ca+ inflow, K+ Outflow, If current • AV node: Ca+ inflow, K+ Outflow, If current • Ventricular muscle: Na+ inflow, K+ outflow, Ca2+ inflow, K+ outflow • Purkinje cells: Na+ and Ca2+ inflow, K+ outflow, K+ outflow, If current
  • 72.
  • 73.
  • 74. Describe the role of ion channels in AP generation and the effects of sympathetic and parasympathetic nerves on AP generation and HR. • Ion channels dictate the speed at which signals propagate through the heart. For instance during phase 0 in nonpacemaker cells, the rate of depolarization depends on the number of activated fast sodium channels. The more sodium channels the more rapidly the cell depolarizes. The faster the cell depolarizes, the faster the adjoining cell will depolarize and thus the more quickly the signal will propagate through the tissue. • Therefore conditions that decrease the availability of fast sodium channels (e.g., depolarization caused by cellular hypoxia), decreases the rate and magnitude of phase 0, thereby decreasing conduction velocity within the heart. • Sympathetic NS act on 훃1-adrenoceptors with norepinephrine to increase conduction velocity and thus heart rate. • Parasympathetic NS act on M2 receptors with acetylcholine to decrease conduction velocity and thus heart rate.
  • 76. Describe the spectrum of ischemic heart disease and its societal effects. • Ischemic heart disease is the leading cause of death in the world among men and women (7 million per year). • Coronary arteries cannot provide enough perfusion to keep up with myocardial demand. This results over time due to atherosclerotic narrowing of the arteries, along with superimposed degrees of plaque changes, thrombosis, and vasospasm. • CAD usually presents as an MI, Angina pectoris (most common), Chronic IHD with heart failure, and sudden cardiac death.
  • 77. Compare Stable angina pectoris, Unstable angina pectoris, Acute MI.
  • 78. What is the most common ECG finding during episodes of stable and unstable angina? • ST Depression
  • 79. Define subendocardial versus transmural infarction and be able to differentiate the ECG segment changes • Subendocardial infarction involves small areas in the subendocardial wall of the left ventricle, ventricular septum, or papillary muscle ; ST depression. • Transmural infarction is associated with atherosclerosis in a major coronary artery; the infarct extends through the thickness of the heart muscle, resulting from nearly complete occlusions; ST elevation.
  • 80. Identify ST elevation and pathologic Q waves and distinguish these findings associated with the anterior, lateral, or inferior walls.
  • 81. Coronary Atheromatous Plaque • Plaque disruption is initiated by the release of substances from inflammatory cells within the fibrous cap. These weakened caps could then either rupture spontaneously or be due to a physical force such as in increase in intraluminal BP. • Following plaque rupture, a thrombus forms due to the activation of platelets (subendothelial collagen), the coagulation cascade, and narrowing of the vascular lumen (vasoconstrictors). • Dysfunctional endothelium  no release of vasodilators such as NO, which would normally oppose the effects of the vasoconstrictors and reduce platelet aggregation. • Destruction of myocytes in acute coronary syndromes quickly impairs ventricular contraction, which is seen as systolic dysfunction. • Ischemia/infarction will also impair diastolic relaxation, which reduces left ventricular compliance and leads to elevated filling pressure.
  • 82.
  • 83. Identify the main difference between stunned and hibernating myocardium. • Stunned myocardium occurs when transient ischemia produces a prolonged (days to weeks) yet reversible period of contractile dysfunction. This may occur in UA (unstable angina) patients or surrounding areas of infarction. • Hibernating myocardium occurs when blood supply is chronically reduced, resulting in chronic contractile dysfunction. It promptly improves when adequate perfusion is restored.
  • 84. Diagnostic Tests for CAD • ECG - look for ST segment elevation/depression and T wave changes - easy but may not “catch” episodes in outpatients • Stress test - provocative exercise or pharmacologic • Nuclear imaging - Te99 or Th201 perfusion studies • Coronary angiography • CT
  • 85. Diabetics may be more likely to have silent angina
  • 86. Describe the two main determinants of coronary blood flow. • The two main determinants of coronary blood flow are (diastolic) pressure and resistance (governed most strongly by a r^4 term).
  • 87. Differentiate endothelial-dependent vasodilation from endothelial-independent vasodilation, and name one compound that works through each mechanism. • Endothelial-dependent vasodilation • NO • Prostacyclin • In atherosclerotic vessels, the endothelium is dysfunctional and fewer of these vasodilating molecules are released. • Endothelial-independent vasodilation • Adenosine • Lactate • Acetate • H+ • CO2
  • 88. Myocardial O2 Supply and Demand
  • 89. Name the primary mechanism by which coronary blood flow is maintained in the presence of moderate epicardial coronary artery stenosis. • Coronary arteries consist of both large, proximal epicardial segments and smaller, distal resistance vessels (arterioles). Atherosclerosis and narrowing almost always occur in the proximal vessels while the arterioles usually stay free of flow-limiting plaques. Thus, blood flow is maintained by the arterioles that can adjust their tone and dilate in response to metabolic needs. However, if the artery narrowing continues, the arterioles will eventually be unable to fully compensate.
  • 90.
  • 91. Correlate the coronary artery that is occluded to the anatomic distribution of an acute myocardial infarction.
  • 92.
  • 93. Complications Ventricular Wall Rupture: 3 to 7 days (associated with macrophages) The anterolateral wall at the midventricular level is the most common site. Gross photo shows tan yellow, soft infarct of 3-5 days duration with rupture of the ventricular wall. Due to excessive phagocytosis from infiltrating macrophages. Often fatal due to cardiac tamponade. Papillary muscle dysfunction: 2-7 days (associated with macrophages) Rupture of a papillary muscle may occur following an MI causing mitral regurgitation. More frequently, postinfarct mitral regurgitation results from ischemic dysfunction of a papillary muscle and underlying myocardium and later from papillary muscle fibrosis and shortening, or from ventricular dilation. Pericarditis: 2-3 days (associated with PMNs) A fibrinous pericarditis usually develops following a transmural infarct is a result of PMN infiltration in reaction to necrosis and myocardial inflammation. Mural thrombus: Within 10 days With any infarct, the combination of a local abnormality in contractility (causing stasis) and endocardial damage (creating a thrombogenic surface) can foster mural thrombosis and potentially thromboembolism. Ventricular Aneurysm: True aneurysms of the ventricular wall are bounded by myocardium that has become scarred. Aneurysms of the ventricular wall are a late complication (4-8 wks) of large transmural infarcts that experience early expansion. The thin scar tissue wall of an aneurysm paradoxically bulges during systole. Complications of ventricular aneurysms include mural thrombus, arrhythmias, and heart failure; rupture of the tough fibrotic wall is rarely a concern.
  • 94. Describe the indications for coronary revascularization, including the need for coronary stenting • For angina patients, coronary revascularization is pursued if: • 1) Angina symptoms do not respond to anti-anginal therapy • 2) Drug therapy results in unacceptable side effects • 3) Patient is found to have high-risk coronary disease for which revascularization is known to improve survival • Persistent angina • Significant stenosis in 1 or 2 coronary arteries • Lower-risk patients with stenosis in all 3 CA
  • 95. Limitations of Stenting • Restenosis: • Neointimal proliferation (migration of smooth muscle cells + ECM production) can occur over the stent, restenosing the vessel • Solution: cover the stent with an anti-proliferative medication that prevents neointimal proliferation. This decreases restenosis by 50%. • Unfortunately, this also slows endothelialization of the stent, resulting in a greater incidence of thrombosis if antiplatelet therapy is discontinued too early. • Give Prasugrel or Clopidogrel for 12 months for drug eluting stents and 1-2 months for bare metal stents • Thrombosis • Stent material is thrombogenic (promotes formation of clots) • Solution: Antiplatelet drug therapy • Give Prasugrel or Clopidogrel for 12 months for drug eluting stents and 1-2 months for bare metal stents
  • 96. Coronary Bypass Surgery • More effective long-term relief of angina than PCI • Improved survival in patients with: • >50% left main stenosis • 3 vessel CAD, especially if LV contractile function is impaired • 2 vessel disease with >75% LAD stenosis • Diabetes patients with multiple vessels involved • More complete revascularization than PCI • Internal Mammary Artery vs. Veins • Veins are vulnerable to accelerated atherosclerosis (50% are occluded after 10 years) and lower 10 yr. patency rate (80%) • Arteries are have higher 10 yr. patency rates (90%) and are more resistant to atherosclerosis. • Specifically, the internal mammary artery appears to be fairly resistant to atherosclerosis • Limitations: stenosis of the grafted vessel due to atherosclerosis, surgical risk v benefits, comparison of benefits of CABG with PCI
  • 97.
  • 98. CAD: PCI versus CABG PCI • Brief hospitalization • Less expensive • Minimally uncomfortable - percutaneous • Restenosis-9% • Stent thrombosis • Clopidogrel CABG • 5-7 days • More expensive • Painful • Usually definitive • Survival advantage • 3VD + reduced LVEF • Left main CAD
  • 99. Exercise Testing • Positive if chest pain or ECG abnormalities are produced. • Exercise test considered markedly positive* if: • 1. Ischemic ECG changes develop in first 3 minutes or persist 5 minutes after exercise stops • 2. Magnitude of ST segment depressions > 2 mm • 3. Systolic blood pressure abnormally falls during exercise (due to ischemia-induced impairment of contractile function) • 4. High-grade ventricular arrhythmias develop • 5. Patient cannot exercise for at least 2 minutes because of cardiopulmonary limitations
  • 100. Coronary Angiography • Coronary Angiography • Most direct means of identifying coronary artery stenosis • Atherosclerotic lesions visualized radiographically following injection of radiopaque contrast material into artery • Procedure generally safe, but small risk of complication due to invasive nature • Reserved for patients whose angina symptoms do not respond to pharmacologic therapy, have an unstable presentation, or when results of noninvasive testing are so abnormal that severe CAD warranting revascularization is likely. • “Gold Standard” for CAD diagnosis, however… • Only provides anatomic information • Clinical significance of lesions depends on pathophysiologic consequences • Standard arteriography does not reveal composition of plaque or vulnerability to rupture
  • 101. Coronary Blood Flow • In coronary arteries, most of blood flow to the myocardium occurs during diastole. • During systole: contraction of myocardium compresses ventricular microvasculature • Blood flow is reduced to the greatest extent within innermost regions of ventricular wall (subendocardium) where compressive forces are greatest • More susceptible to injury in ischemic events, CAD, reduced aortic pressure • Blood flow reaches peak in early diastole – where compressive forces removed • Aortic pressure during diastole thus is most crucial for perfusing coronaries • In left ventricle – mechanical forces affecting coronary flow are greatest • Due to the higher pressures associated • Right ventricle and the two atria show some effects of contraction and relaxation of blood flow • However, it’s much less apparent than the L ventricle
  • 102. Explain how arterio-venous O2 difference and O2 extraction in the heart is unique when compared with other body organs. • Unlike most tissues, the heart cannot increase oxygen extraction on demand, because in its basal state it removes almost as much oxygen as possible from its blood supply. • So any additional oxygen requirement must be met by an increase in blood flow. • Autoregulation of coronary vascular resistance is most important mediator of this process • Factors regulating coronary vascular resistance: • Accumulation of local metabolites • Endothelium-derived substances • Neural innervation
  • 103. Describe what is meant by coronary vascular reserve and the role of collateral blood vessels. • Coronary flow reserve is the maximum increase in blood flow through the coronary arteries above normal resting volume. • The CFR is reduced in coronary artery disease, i.e. there is reduced vasodilator reserve • When O2 delivery to heart limited by disease, collateral vessels arise through angiogenesis • Process stimulated by chronic stress (hypoxia, exercise training, etc.) • Collateralization increases myocardial blood supply by increasing the number of parallel vessels • Reduces vascular resistance within myocardium
  • 104. List the four major coronary arteries and identify the structures they supply. • Left main coronary artery • Left anterior descending artery • Circumflex Artery • Right main coronary artery • Left coronary artery supply • Left ventricle • Left atrium • Interventricular septum • Right Coronary Artery supply • Right Atrium • Right ventricle • SA node • AV node • Interventricular septum
  • 106. Identify the factors that regulate myocardial oxygen consumption and myocardial oxygen delivery. • The myocardium has one of the highest O2 extraction ratio of all body organs. In a normal human being at rest the heart consumes 11 % of total body oxygen but receives only about 4 % of the cardiac output as coronary blood flow. • • Supply of oxygen depends upon: • · Oxygen content of blood (systemic oxygenation and hemoglobin) • · Coronary blood flow (perfusion pressure and coronary vascular resistance) • • Myocardial oxygen demand depends upon: • · Wall stress • · Heart rate • · Contractility • • O2 demand is increased by ↑ HR, ↑ heart contractility, ↑ preload, ↑ afterload, ↑ ejection time. • O2 supply is reduced by ↑ HR, ↑ preload, ↓ artery diameter (atherosclerosis).
  • 107. Stable Angina • Angina is an imbalance between coronary blood flow and cardiac O2 consumption leading to ischemia • Stable angina (classic angina, or angina of effort; most common form): • Ischemia is caused by stable coronary artery narrowing (atheromatous plaque) • Predictable pain on exertion or psychological stress. • Unchanged in severity, frequency, and duration over weeks to months
  • 109. Diagram the neural pathway involved in anginal pain. • The myocardium is innervated by chemosensitive sensory afferent that are activated by products of hypoxia such as adenosine (a breakdown product of ATP), acidification (caused by anaerobiosis) and the release of autacoids such as serotonin and prostaglandins E. • The peripheral axons of these sensory afferents travel within the sympathetic chain. Their cell bodies are located in dorsal root ganglia (DRGs) at thoracic level. The central process contact spinal interneurons located in the dorsal horn of the spinal cord which in turn activate spino-reticular and spinothalamic pathways. The spino-reticular pathway activate the vasomotor center and therefore increases sympathetic tone to the heart potentially causing further ischemic damage by increasing oxygen demand. • Activation of the spinothalamic pathway causes anginal pain (except in people with silent angina). If pain occurs, this causes further increase in SNA via descending pathways through the medullary vasomotor center. • Anginal pain is referred to the neck shoulder and arm region because nociceptive afferents that originate from these regions of the bodies and cardiac nociceptors converge on the same spino-thalamic neurons.
  • 110. Recall the mechanism whereby coronary blood flow is coupled to myocardial workload. • Increased contractility/HR increases ATP breakdown leading to increased local concentration of adenosine, stimulating vasodilation and increased coronary flow to meet the O2 demands with the increased myocardial workload.
  • 111. Explain why pain, anxiety or exercise can exacerbate cardiac ischemia. • Pain, anxiety or exercise cause release of chemical mediators: NE, 5-HT that activate the sympathetic nervous system. • (1) The increase in sympathetic activity (β1 receptors) and the decrease in parasympathetic activity produce an increase in HR. • (2) The increase in sympathetic activity (β1 receptors) produces an increase in contractility and a resulting increase in SV. • Together, the increases in heart rate and stroke volume produce an increase in cardiac output. • This leads increased O2 demand will exacerbate cardiac ischemia. • Furthermore, pts with cardiac ischemia often have narrowed coronary arteries (atherosclerosis) so they have dysfunctional endothelium (less vasodilatory/antithrombogenic properties) along with increased resistance so that it is hard for the heart to keep up enough of an O2 supply with the increased workload. • Stimulation of alpha receptors by catecholemines released during stress, exercise and pain can lead to vasconstriction and without enough local metabolites like endothelial NO to offset that with vasodilatiion (dysfunctional endothelium) one could see less sympatholysis.
  • 112. Identify the main goals of therapy for patients with stable angina. • Decrease frequency of anginal attacks • Prevent acute coronary syndromes • Prolong survival
  • 113. MOA of nitroglycerin and isosorbide mononitrate on vascular smooth muscle and identify which blood vessels are preferentially targeted by low doses • Nitroglycerine is converted to Nitric oxide which activates guanylate cyclase leading to an increase in cGMP in smooth muscle. This leads to dephosphorylation of Myosin Light Chains which regulate the contractile state of smooth muscle and lead to vasodilation. • Nitroglycerin has a fast onset (2-5 minutes) with sublingual administration and a short duration (30 min). Low dose – relaxation of great veins • It is used for acute attacks of angina • Isosorbide Mononitrate: Organic nitrate that causes vasodilation through enzymatic conversion of sulfhydryl groups to nitric oxide • Slower onset, longer duration • Used for chronic treatment
  • 114. Explain why nitroglycerin must be administered sublingually while isosorbide dinitrate or mononitrate is given orally. • Rapid onset of action and are useful as prophylaxis • Sublingual admin leads to fast absorption directly into the systemic circulation, thereby avoiding delay inherent to intestinal absorption • First pass metabolism (degradation) by the liver → nitroglycerin cannot be taken orally because it would be entirely degraded by the liver • Very lipid soluble compound that is well absorbed by the mucosa of the tongue and mouth • Spray more stable → in this formulation it stays active for years while tablets take up moisture which degrades the nitroglycerine within a month
  • 115. Explain why NO donors and PDE5 inhibitors (e.g. sildenalfil) should not be co-administered. • The combination of a PDE5 inhibitor (sildenafil (Viagra), etc) and nitrates  extreme hypotension • If PDE5 is inhibited cGMP cannot be converted to GMP, so much more cGMP is produced leading to extreme vasodilation
  • 116. Nitrate tolerance • The main limitation to chronic nitrate therapy is the development of drug tolerance • Overcome this with: • 1) a nitrate-free interval for a few hrs (8-12 hrs each day) • 2) add drugs that reduce the requirement for nitrates (such as B-blockers or Ca++ channel blockers) • Mechanism theories include: • 1) Sulfhydryl hypothesis  depletion of SH groups need for conversion to NO • 2) Neurohormonal hypothesis  reflex increase in vasoconstrictor hormones (NE, tissue RAS, endothelin) • 3) Free radical hypothesis  free radicals destroy NO
  • 117. Explain the rationale for the potential benefit of combining a beta-blocker with a nitrate in treating stable angina.
  • 118. Statins Mechanism Competitively inhibit HMG-CoA reductase: (1) Decreases intracellular cholesterol  induces SREBP  increases expression of LDL-R (2) VLDL and IDL are cleared more rapidly due to cross-recognition with hepatic LDL-R (3) Hepatic VLDL production falls due to reduced cholesterol availability  reduced LDL and triglycerides Modify platelets and endothelium (e.g., enhanced NO synthesis) Suppress inflammation Effects Decreases LDL 18-55% Decreases TG 7-30% Increases HDL 5-15% (unclear mechanism) Side Effects Myopathy (increased w/ niacin, fibrates), hepatotoxicity, drug interactions (CYP3A4 inhibition: macrolides, azoles, HIV protease inhibitors)
  • 119. Niacin Mechanism Decreases lipolysis in adipose tissue  less FAs available for TG synthesis in liver Decreases VLDL synthesis, so less LDL Increases HDL by decreasing hepatic removal of HDL Effects Decreases LDL 5-25% Increases HDL 15-30% Decreases TGs 20-50% Side Effects Cutaneous flushing (due to prostaglandins; take aspirin), GI (nausea, PUD), hepatotoxicity, insulin resistance and hyperglycemia (caution w/ diabetics), gout (raises serum uric acid levels), myopathy (increases w/ statin)
  • 120. Fibrates Mechanism Activate PPARα-RXR  (1) Enhanced oxidation of FAs in liver and muscle  decreased TG levels  decreased VLDL (2) Increased expression of LPL (3) Increased rate of HDL-mediated reverse cholesterol transport (due to apo AI transcription) Effects Decreases LDL 5-20% Increases HDL 10-20% Decreases TGs 20-50% *Larger decreases in TGs and increases in HDL than statins. Side Effects GI (dyspepsia, abdominal pain, diarrhea), cholesterol gallstones, myopathy (increased w/ liver and kidney dysfunction; worse w/ statins), augment effects of oral hypoglycemic drugs (avoid in diabetes)
  • 121. List the major side effects of antianginal medications, including which drugs when combined have an increased risk of SEs • Nitrates: headache, lightheadedness, hypoTN, palpitations, nausea, dizziness, reflex sinus tachycardia (BP decrease causes heart to compensate and beat faster). • DO NOT ADMINISTER with PDE5 inhibitors. • B-blockers: bronchospasm  AVOID in pts with COPD, reduced HR  contraindicated in pts with bradycardia, fatigue, sexual dysfunction, may worsen diabetic control and can mask tachycardia and other signs that indicate hypoglycemia  *** Diabetic pts • Calcium channel blockers: associated with an increased incidence of MI and mortality, Headache, flushing, decreased LV contraction (esp with Verapamil and Diltiazem) , pedal edema (esp with Nifedipine and Diltiazem), constipation (esp with Verapamil) • Combining a B-blocker with a nondihydropyridine Ca++ channel blocker (Verapamil or Diltiazem): negative chronotropic (“changing HR”) effect that can cause excessive bradycardia, combined with a negative inotropic effect, could precipitate heart failure in pts with LV contractile dysfunction • Ranolazine: dizziness, headache, constipation, nausea
  • 122. Describe the mechanism of Prinzmetal (variant) angina • Prinzmetal’s (variant) angina is ischemia due to focal coronary artery spasm. • Mechanism: Patient has intense vasospasm. → Reduced coronary oxygen supply → Prinzmetal’s Angina. • The cause of the vasospasm is unknown, but it is thought that it may be caused by increased sympathetic activity + endothelial dysfunction. • Patient presentation: Typical anginal discomfort, usually at rest rather than upon exertion. The pain is often very severe. • ECG findings: ST segment elevations during the intense vasospasm. ST segment elevation signifies injury. • Good antianginal drugs for this type of angina: Verapamil & Diltiazem • Beta Blockers are contraindicated as they may worsen the condition
  • 123. Describe the typical ECG findings during an episode of stable angina. • ST segment and T wave changes • Transient horizontal or downsloping ST segment depressions • T wave flattening or inversions • Occasionally ST segment elevations are seen, suggesting more severe transmural myocardial ischemia • In contrast to an acute MI, ST deviations caused by angina quickly normalize with resolution of the patient’s symptoms.
  • 124. Describe the findings of a positive exercise ECG treadmill stress test. • The test is considered positive if the patient’s typical chest discomfort is reproduced or if ECG abnormalities consistent with ischemia develop (ex: >1 mm horizontal or downsloping ST segment depressions) • The test is considered markedly positive if one or more of the following signs of ischemic heart disease occur: • Ischemic ECG changes develop in the first 3 minutes of exercise OR persist 5 minutes after exercise has stopped • The magnitude of ST segment depressions is >2 mm • The systolic blood pressure abnormally falls during exercise (i.e. resulting from ischemia induced impairment of contractile function) • High grade ventricular arrhythmias develop • The patient cannot exercise for at least 2 minutes because of cardiopulmonary limitations
  • 125. Recall the effect of a 60% epicardial coronary artery stenosis on resting coronary blood flow versus its effect on maximal coronary flow. • This is pointing to the fact that if an epicardial coronary artery is stenosed to a level of 60%, this will limit the maximum amount of blood that can flow through a coronary artery (and therefore may lead to angina when you are exercising or exerting yourself), but it will have no effect on the resting coronary blood flow, which is much less.
  • 126. Explain why nuclear cardiac imaging and echocardiography are sometimes performed in conjunction with exercise stress testing. • Some patients have baseline ST segment abnormalities or T wave abnormalities (particularly LVH with strain). For these patients, the ECG findings provided by the exercise stress test aren’t very useful. • Also, the exercise stress test can yield ambiguous results - this is especially of concern when there is a high clinical suspicion of ischemic heart disease. • Exercise stress testing in conjunction with nuclear cardiac imaging or echocardiography increases sensitivity and specificity.
  • 127. Explain the most common reasons that percutaneous revascularization (PCI) is offered to patients with stable angina. • Many patients with stable angina can be managed with pharmacologic therapy alone. • PCI is offered to patients with stable angina if: • The patient’s symptoms of angina do not respond adequately to antianginal drug therapy • Unacceptable side effects of medications occur • The patient has high risk coronary disease for which revascularization is known to improve survival
  • 128. Name one criterion by which patients are selected for CABG instead of PCI. • CABG is good for: • >50% left main stenosis • 3 vessel CAD, especially if LV contractile function is impaired • 2 vessel disease with >75% LAD stenosis • Diabetes patients with multiple vessels involved • PCI is good for: • Patients with persistent episodes of angina and significant stenoses in one to two coronary arteries • Some lower risk patients with three-vessel disease
  • 130. Describe the pathophysiologic events that change a stable atherosclerotic plaque into the unstable plaque of ACS • Increasing size and protrusion of lipid core  mechanical stress focused on plaque border • Local accumulation of foam cells and T lymphocytes releasing MMPs  increases degradation of ECM. • Thin fibrous cap  vulnerability to rupture • Rupture of plaque  exposure of procoagulants  thrombosis • Causes occlusion and infarction • Endothelial dysfunction prevents release of endogenous vasodilators (NO, prostacyclin) which also normally inhibit platelets, thus impairing protective mechanisms against thrombosis.
  • 131. Compare and contrast the pathophysiologic and clinical features of unstable angina, non-STEMI and STEMI.
  • 132. Recognize the non-atherosclerotic causes of an acute coronary syndrome.
  • 133. Describe the functional alterations impairing contractility and compliance. • Systolic dysfunction - destruction of functional myocardial cells leading to impaired ventricular contraction • Hypokinetic: a localized region of reduced contraction • Akinetic - a segment that does not contract at all • Dyskinetic - a segment that bulges outward during contraction of the remaining functional portions of the ventricle • Diastolic dysfunction - compromise of the left ventricle when ischemia/infarction causes elevated ventricular filling pressures
  • 134. Define and distinguish the terms “stunned myocardium”, “ischemic preconditioning” and “infarct expansion” • Stunned myocardium - prolonged but reversible period of contractile dysfunction after a period of transient ischemia. The tissue prolongs systolic dysfunction even after restoration of blood flow. Contractile force is regained days to weeks later. If the tissue is simply stunned rather than necrotic, its function will recover. • Ischemic preconditioning - brief ischemic insults to a region of myocardium that make that region more resistant to subsequent episodes. Thus, patients who have an MI after recent anginal pain often have less morbidity and mortality than those with an “out-of-the-blue” MI. The conditioning may be triggered by substances released during ischemia like adenosine and bradykinin. • Infarct expansion - in an early post-MI period, the affected ventricular segment enlarges without additional myocyte necrosis - occurs by thinning and dilatation of the necrotic zone from “slippage” between muscle fibers. • The increase in ventricular size 1) augments wall stress, 2) impairs systolic contractile function, and 3) increases the likelihood of aneurysm formation
  • 135. Cellular Changes of Acute MI • Occluded coronary vessel  falling O2 levels  switch from aerobic to anaerobic metabolism  lactic acid accumulation  lowered pH (Metabolic Acidosis) • The decrease in high-energy phosphates like ATP interfere with Na+/K+ ATPase  elevation in intracellular Na+ and extracellular K+ • Rising intracellular Na+  cellular edema • Membrane leakage and rising extracellular K+  altered transmembrane electrical potentials and predisposition to arrhythmias • Intracellular Ca2+ accumulation activates degradative lipases and proteases and contributes to final pathway of cell destruction. • Metabolic changes decrease function within 2 minutes of an occlusive thrombosis. • Without intervention, irreversible cell injury ensues within 20 minutes • Proteolytic enzymes leak across membrane and damage myocardium • Release of macromolecules into circulation (Troponin @ 4 hours)
  • 136. Compare and contrast the use and time course of troponin and CPK-MB in diagnosis an acute MI • Troponins • The specific troponins used for the diagnosis of MI are cTnI and cTnT, because these are the cardiac forms of troponin I and are also structurally unique, and thus easier to assay. • These markers tend to increase 3-4 hours after onset of discomfort, peak between 18-36 hrs and may be present for up to 2 weeks. • CPK-MB • Isoenzyme of creatine kinase that exists in the heart - CPK-MB. • The measurement for CPK-MB is calculated by the ratio of CPK-MB: total CPK. • Values of >2.5% usually indicative of cardiac injury • Levels of CPK-MB rises between 3-8 hrs following infarction, peaks at 24 hrs and goes back to normal after 48-72 hrs. • This makes it a useful indicator for REINFARCTS. • Neither of these markers are good for early diagnosis of MI, since they take a few hours to peak. In early situations, ECG and history are most important.
  • 137.
  • 138. Do not use Fibrinolytic treatment regimens for NSTEMI
  • 139. Name at least two fibrinolytic agents and explain the benefits, limitations and major risks of thrombolytic therapy. • Alteplase, tPA • Reteplase, rPA • Tenecteplase - TNK-tPA • Very effective in lysing the intracoronary thrombi found in STEMI. Patients who receive FT quickly (within 2 hours of onset of symptoms) have half the rate of mortality of STEMI pts who get it after 6 hrs • Fibrinolytic therapy does not benefit patients suffering from UA or NSTEMI. • Bleeding is the most common complication of fibrinolytic therapy, patients who require effective fibrin clotting are contraindicated for this therapy. That includes post-op patients, those with a bleeding disorder or recent stroke.
  • 140. Define the term “primary percutaneous coronary intervention” and explain the benefits, limitations and major risks • Angioplasty + stenting of the vessel • Medications given during PCI: Aspirin, Heparin, IV GP IIb/IIIa receptor antagonist • *may substitute Direct Thrombin Inhibitor (e.g. Bivalirudin) for Heparin + GP IIb/IIIa antagonist combo • If receiving a stent → Oral Thienopyridines (e.g. Clopidogrel) given to reduce risk of ischemic complications & stent thrombosis • Clopidogrel or Prasugrel given for >12 months after stent placed • Benefits: • Treat patients with contraindications to Fibrinolytic therapy or unlikely to do well with fibrinolysis (e.g. late presentation to hospital - more than 3 hrs with symptoms, in cardiogenic shock) • Treat patients initially treated with fibrinolytic therapy without adequate response (e.g. ST segment elevations) • In comparison to Fibrinolytic Therapy: • Greater survival & Lower rates of reinfarction and bleeding • Preferred method IF performed by experienced operator within 90 mins of arriving
  • 141. TIMI Risk Score • Patients with “most concerning clinical features” → identified by risk assessment algorithms (higher scores: ≥ 3)TI • Age > 65 years old • ≥ 3 Risk factors for coronary artery disease • Known Coronary Stenosis of ≥ 50% by prior angiography • ST segment deviations on ECG at presentation • ≥ 2 anginal episodes in prior 24 hrs • Use of Aspirin in 7 days prior • Elevated serum troponin or CK-MB
  • 142. Identify the most important predictor of post-MI outcome, and describe how to risk-stratify a patient after acutely treating their myocardial infarction. • Most important predictor of post-MI outcome - LV Dysfunction • Identify patients at high risk for complications: • Exercise treadmill testing • Attention to underlying cardiac factors • Smoking • Hypertension • Diabetes • LV ejection fraction of less than or equal to 30% after MI  high risk of sudden cardiac death • Prophylactic placement of implantable cardioverter-defibrillator recommended
  • 143.
  • 144. Ejection Fraction of <30% after MI is high risk for sudden cardiac death and suggests prophylactic placement of ICD.
  • 146. Know the electrode placements and polarities for a 12‐lead electrocardiogram
  • 147.
  • 149. Standard values for EKG print out
  • 150. Correlate tracing to electrical state of heart
  • 151. Systematic Approach • Rate • Rhythm • Axis • Intervals • Hypertrophy • Ischemia • Special Changes
  • 152. RATE
  • 153. A rate of 60-100 is normal <60 is bradycardia and >100 is tachycardia
  • 154. Method 1 for Determining Rate
  • 155. Method 2 for Determining Rate
  • 156. Method 3 for Determining Rate
  • 157. RHYTHM
  • 158. Criteria for Normal Rhythm • A P wave morphology P wave (atrial contraction) precedes every QRS complex • A QRS complex follows every P wave • The rhythm is regular, but varies slightly during respirations • The rate ranges between 60 and 100 beats per minute • The P waves maximum height at 2.5 mm in II and/or III • The P wave is positive in I and II, and biphasic in V1
  • 159. Is there a QRS after every P? • NO • If rate < 100: • a) 2 block type I ("Wenkebach," gradually lengthening PR until one beat is dropped) • b) 2 block type II (dropped beat without change in PR), or • c) 3 AV block (no correlation between P and QRS) • If rate >100: • atrial or nodal tachycardia (SVT) or atrial flutter, both with block.
  • 160. Is there a P before every QRS? • NO • a) if a single slow beat: escape beat • atrial if different P • nodal if no P • ventricular if QRS>0.12 • b) if a slow rhythm: escape rhythm • nodal if no P at rate 50-60 • ventricular if QRS>0.12 & rate <40 • c) if a single fast beat: premature beat • PAC if different P • PJC if no P • PVC if QRS>0.12
  • 161. Tachycardic • a) if wide (>0.12) and rate >120 then ventricular tachycardia until proven otherwise • b) if narrow and regular then atrial (with preceding P) or AV nodal (no P) tachycardia • c) if irregularly irregular then either • 1) atrial fibrillation (no P waves and coarse baseline) • 2) multifocal atrial tachycardia (MAT, three different P wave morphologies).
  • 162. AXIS
  • 163. Define mean electrical vector (axis) of the heart and give the normal range.
  • 164. Determine the mean electrical axis from knowledge of the magnitude of the QRS complex in the standard limb leads. • Inspect limb leads and determine QRS that is most isoelectric, the mean axis is perpendicular to that lead • Inspect the lead that is perpendicular to the isoelectric complex, if the QRS is primarily upward, then the mean axis points towards the (+) pole of the lead.
  • 167. Intervals • PR • Measure of the health of the AV node and bundle of His • Normal: < 0.2 s (1 big box) • Pathology: Prolonged interval: AV blocks 1,2,3 • QRS • Measure of the health of the His-Purkinje system • Normal: < 0.12 s (3 small boxes) • Pathology: Conduction delay: LBBB,RBBB, fascicular blocks • QT • Measure of repolarization • Normal: < 0.45 s • Pathology: Long QT syndrome, electrolyte imbalance, ischemia
  • 168. First Degree AV Block • Prolonged PQ Interval >.2s
  • 169. Second Degree AV Block-Wenkebach • In second degree AV block type I, the PQ interval prolongs from beat to beat up until the drop-out of one QRS complex. The characteristics of a Wenkebach block: • QRS complexes cluster • The PQ interval prolongs every consecutive beat • The PQ interval that follows upon a dropped beat is the shortest. • The amount of block decreases during exercise
  • 170. Second Degree AV Block – Mobitz II • In second degree AV block type II, beats are dropped irregularly without PQ interval prolongation. • As the drop out of beats is irregular, no clustering of QRS complexes can be seen as in second degree block type I. • Second degree AV block type II marks the starting of trouble and is a class I pacemaker indication
  • 171. Third Degree AV Block • Third degree AV block is synonymous to total block: absence of atrioventricular conduction. The P-waves and QRS complexes have no temporal relationship: AV dissociation. The ventricular rhythm can be nodal, idioventricular or absent. • During third degree AV block the blood supply to the brain can insufficient, leading to loss of consciousness. Adams Stokes attacks attacks are attacks of syncope or pre-syncope in the setting of third degree AV block.
  • 172. QRS Interval • Measure QRS interval, if > 0.12 then Look at V1 for LBBB and RBBB • Left bundle branch block (downgoing) • Right bundle branch block with (bunny ears and upgoing)
  • 173. LBBB
  • 174. RBBB
  • 175. LAFB • Left Axis Deviation • qR in the lateral leads • rS in the inferior leads • Mild QRS widening
  • 176. LPFB • Right Axis Deviation • rS in lateral leads • qR in inferior leads • Mild or no QRS widening
  • 177. QT Interval • Bazett’s Formula • QTc = [QT Interval] / √[R-R interval] • Important due to R on T risks • Acquired • Medications • Electrolyte abnormalities • Ischemia • Hypothermia • Genetic • Sodium Channel abnormalities • Potassium Channel abnormalities • High risk for ventricular fibrillation
  • 178.
  • 179. Brugada Syndrome • ST elevation ≥2 mm and a coved type ST segment followed by a negative T wave. This morphology must be present in >1 right precordial lead (V1-V3).
  • 180. Wolff Parkinson White • Short PR interval with delta wave
  • 183. Differentiate left atrial enlargement from right atrial enlargement on an ECG • Right Atrial Enlargement • Lead II – P wave > 2.5 mm • Lead V1 – Biphasic p wave with upright deflection larger • Left Atrial Enlargement • Lead II – The P wave is broader • P mitrale • Lead V1 – Biphasic p wave with terminal component larger
  • 187. LEADS II AND V1 ARE MOST PARALLEL TO ATRIAL DEPOLARIZATION BEST AREA TO VIEW P WAVE
  • 188. ECG of ventricular hypertrophy
  • 189. Left Ventricular Hypertrophy • R in V5 or V6 + S in V1 >35 mm
  • 190. Look for ST depression STRAIN
  • 191. Right Ventricular Hypertrophy • QRS duration < 120ms • Right heart axis (> 110 degrees) • Dominant R wave in V1
  • 193. Infarction • Ischemia: inverted T waves or ST depression • Injury: ST elevation • Necrosis: Pathologic Q waves
  • 194. Myocardial Ischemia • ST depression and T-wave changes. • New horizontal or down-sloping ST depression >0.05 mV in two contiguous leads; and/or T inversion ≥0.1 mV in two contiguous leads with prominent R-wave or R/S ratio ≥ 1 • ST elevation • New ST elevation at the J-point in two contiguous leads with the cut-off points: ≥0.2 mV in men or ≥ 0.15 mV in women in leads V2–V3 and/or ≥ 0.1 mV in other leads.
  • 196. Myocardial Necrosis • Pathologic Q waves – irreversible injury • Localize based upon which arteries are involved • Any Q wave > 1 small box in duration and more than 1/3 the height of the R wave in 2 contiguous leads.
  • 197. Predict which coronary artery is affected in a patient experiencing an acute myocardial infarction.
  • 198. Anterior Hemiblock • LAD occlusion • Normal or slightly widened QRS • Q1S3
  • 199. The only upside down QRS complex allowed is in AVR. Do not assess Q waves in AVR.
  • 200. ST segment depression in NSTEMI is nonlocalizing to specific arteries
  • 201. Be careful about diagnosing an infarct in the presence of LBBB.
  • 203. Outline the phases of the cardiac cycle and describe which phases demarcate systole and diastole. • There are 7 phases in the cardiac cycle. • Systole: phases 2-4 • ventricular contraction and ejection • Diastole: phases 5-7 and 1 • ventricular relaxation and filling
  • 204.
  • 205. The Cardiac Cycle • 1. During diastole the mitral valve is open so LA and LV have equal pressure • 2. Late diastole, LA contraction causes small rise in pressure (a wave) • 3. Systolic contraction, LV pressure rises and MV closes when LV exceeds LA pressure (S1) • 4. When LV pressure exceeds aortic pressure, AV valve opens (silent) • 5. Ventricle relaxes, pressure drops below aorta, AV closes (S2) • 6. LV pressure falls below and MV opens (silent)
  • 206. Atrial Waveform • a wave: Increase due to atrial contraction. • x descent: Fall in atrial pressure due to end of atrial contraction. • c wave: Increase due to bulging of AV valves back into atrial chambers. • x’ descent: Fall in atrial pressure after ‘c wave’ due to rapid ventricular ejection. • v wave: Peak due to continuing venous return just prior to AV valves opening. • y descent: Rapid fall in atrial pressure after ‘v wave’ due to opening of AV valves.
  • 207. Systole occurs approximately between the S1 and S2 heart sounds
  • 208. Derive the stroke volume and left ventricular ejection fraction from the left ventricular end-systolic and end-diastolic volumes. • SV= LVEDV-LVESV • EF = SV/LVEDV • Normal ≥ 55% • The left ventricular ejection fraction is the percentage of blood that leaves the left ventricle with each contraction.
  • 209. Diagram the timing of the s1, s2, s3, and s4 heart sounds to the left ventricular pressure curves and identify the mechanical events that cause each sound. • S1 = Mitral valve closes • S2 = Aortic valve closes • S3 = Mitral valve opens. This can be normal in children and pregnant women but is pathological in adults. Systolic defect associated with ventricular dilation. • S4 = Pathological. Blood is being forced against a stiff ventricle. Ex: Left ventricular hypertrophy. Diastolic defect.
  • 210.
  • 211. Describe the relative contribution of passive and active left ventricular filling and the effects of heart rate and sympathetic activation on this ratio. • Active ventricular filling is associated with atrial contraction, while passive ventricular filling depends on venous return and occurs before atrial contraction. • At Rest: Active filling accounts for approximately 10% of total left ventricular filling. • At High Heart Rates (Exercise): Active filling accounts for up to 40% of left ventricular filling. This increased contribution results from two factors: • Increased heart rate leads to shortened periods of diastolic filling → Reduced amount of blood entering the ventricle during passive filling. • Sympathetic nerve activation increases the force of atrial contraction → Increased amount of blood entering the ventricle during active filling. • This phenomenon is known as “atrial kick” • Clinical Aside: In atrial fibrillation, atrial contraction does not contribute to ventricular filling. This leads to inadequate filling that is exacerbated during physical activity.
  • 212. State the mean right and left atrial pressures and peak and mean right and left ventricular pressures • Mean Right Atrial Pressure: 4 mmHg (average) • Mean Right Ventricular Pressure: 25 mmHg (systolic) and 4 mmHg (diastolic) • Mean Left Atrial Pressure: 8 mmHg (average) • Mean Left Ventricular Pressure: 120 mmHg (systolic) and 8 mmHg (diastolic) • The left heart has higher average pressures because the left ventricle must eject blood into the entire systemic circulation. The right heart has lower average pressures because it is responsible for ejecting blood only to the lungs.
  • 213.
  • 214. Dicrotic Notch in Pressure Tracing • A very brief and transient increase in aortic pressure that corresponds with closure of the aortic valve at the conclusion of systole.
  • 215. Define cardiac output and cardiac index and describe their relationship • Cardiac Output: CO = SV X HR • Cardiac Index (CI) = CO / BSA • Cardiac index is a variation of cardiac output that normalizes for the size of the individual.
  • 216. Describe the relationship between SV and HR and their relative influence on cardiac output. • CO = HR X SV is the basic relationship. However, since changes in heart rate can affect stroke volume, changes in heart rate do not correspond to an exactly proportional change in cardiac output. • Examples: • As heart rate increases through pacemaker stimulation, ventricles have less time to fill with blood during diastole. Less ventricular filling corresponds with a decreased stroke volume. • When heart rate increases by 2X due to pacemaker stimulation alone, cardiac output increases less than 2X. • When heart rate increases by 2X due to exercise, cardiac output increases more than 2X.
  • 218. Know the symptoms and signs acute coronary syndrome • Retrosternal pressure radiating to neck, jaw or left shoulder and arm (C7-T4); more severe and lasts longer than previous anginal attacks • Sympathetic response: Diaphoresis, tachycardia, cool, clammy skin • Systolic dysfunction: dyspnea • Ventricular noncompliance: S4 and S3 heart sounds • Inflammation: Fever • Serum Markers: Increased troponin and CK-MB • ECG: ST depression or elevation, inverted T wave, Q wave
  • 219.
  • 220.
  • 221. ECG of Unstable Angina and NSTEMI
  • 222. ECG findings of an acute MI
  • 223. Localize the site of an infarction and know the likely infarct related artery based on an ECG.
  • 224. Ventricular Fibrillation and Tachycardia • Rapid, disorganized electrical activity of the ventricles • Most fatal before arrival at hospital • If present 48 hours after MI, then typically reflects severe left ventricular dysfunction and is associated with high mortality rates • If it presents <48 after MI, prognosis is much better and often due to transient electrical instability
  • 225. Atrial Fibrillation • Result from atrial ischemia or atrial distension second to LV failure
  • 226. Sinus Tachycardia and Bradycardia • Bradycardia: due to excessive vagal stimulation or SA nodal ischemia in the setting of inferior wall MI • Tachycardia: due to pain, anxiety, heart failure, drug administration or intravascular volume depletion • Can exacerbate ischemia
  • 227. Complete Heart Block • May result from ischemia or necrosis of conduction tracts • May develop transiently from increased vagal tone
  • 228. Explain the difference between ST elevation MI (STEMI) and non STEMI and discuss why a timely diagnosis is most important for a STEMI. • A STEMI involves the complete occlusion of one of the coronary arteries and leads to severe ischemia • ST elevation localizes on ECG based on which artery is involved • A timely diagnosis is extremely important for as irreversible damage to myocytes begins to occur after about 20 minutes. • Changes seen later in ECG such as inversion of the T wave and abnormal Q waves can also be avoided following successful treatment if MI is recognized.
  • 229.
  • 230. Fibrinolysis vs. primary PCI for treatment of an acute MI. • NSTEMI: With a NSTEMI, fibronolysis is never used as patients do not benefit from this therapy. The decision whether to proceed with PCI is based upon a patient’s TIMI score. An early invasive strategy is recommended in patients with higher scores (≥3). If an early invasive approach is adopted, the patient should undergo angiography within 24 hours. • STEMI: In contrast to UA and NSTEMI, the culprit artery in STEMI is typically completely occluded, and therefore, the major focus of acute treatment is to achieve prompt reperfusion of the jeopardized myocardium using either fibrinolytic drugs or percutaneous coronary mechanical revascularization. • Primary PCI is usually the preferred reperfusion approach in acute STEMI, if the procedure can be performed by an experienced operator within 90 minutes of hospital presentation. • In addition, primary PCI is preferred for patients who have contraindications to fibrinolytic therapy or are unlikely to do well with fibrinolysis, including those who present late (>3 hours from symptom onset to hospital arrival) or are in cardiogenic shock. Furthermore, “rescue” PCI is recommended for patients initially treated with fibrinolytic therapy who do not demonstrate an adequate response.
  • 231. Describe the rationale and the indication for adjunctive therapies for the management of the ACS • Focus of treatment for STEMI, UA and NSTEMI consists of anti-ischemic medications to restore the balance between myocardial oxygen supply and demand and anti-thrombotic therapy aimed at preventing further growth, and facilitating resolution of the underlying occlusive coronary thrombus. • Beta blocker: lower heart rate • Propanolol • Metoprolol, Atenolol, esmolol, acebutolol • Aspirin/Clopidogrel: antiplatelet • Aspirin • Clopidogrel • Prasugrel • Ticagrelor • ACE Inhibitors: reduces LV remodeling • Statins: treats atherosclerosis
  • 232. Define "cardiogenic shock" and explain the clinical manifestations of this syndrome. • Cardiogenic shock is a condition of severely decreased cardiac output and hypotension (systolic blood pressure < 90 mm Hg) with inadequate perfusion of peripheral tissues that develops when >40% of the LV mass has infarcted (after MI). • Chest pain/pressure, tachypnea, tachycardia, weak pulse, skin that is pale/blotchy/sweaty, lightheaded, disoriented, syncope, coma, decreased urination.
  • 233. List the four types of "non-cardiogenic shock" and be able to explain the differences from cardiogenic shock. • 1. Hypovolemic shock accompanies significant hemorrhage, or fluid loss from severe burns, chronic diarrhea, or prolonged vomiting. The direct consequence of hypovolemia is inadequate cardiac filling and reduced stroke volume  reduced CO. • 2. Anaphylactic shock is a result of an allergic reaction. This immediate hypersensitivity reaction is mediated by histamine, prostaglandins, leukotrienes, bradykinin that results in substantial arteriolar vasodilation, increases in microvascular permeability, and loss of peripheral venous tone. These combine to reduce both total peripheral resistance and cardiac output. • 3. Septic shock is also caused by profound vasodilation but specifically from substances released into the circulating blood by infective agents • 4. Neurogenic shock is produced by loss of vascular tone due to inhibition of the normal tonic activity of the sympathetic vasoconstrictor nerves and often occurs with deep general anesthesia or in reflex response to deep pain associated with traumatic injuries. It may also be accompanied by an increase in vagal activity, which significantly slows the cardiac beating rate. This type of shock is often referred to a vasovagal syncope.
  • 234. Explain the cardiovascular alterations occurring in shock (both compensatory and decompensatory). • Compensatory: Increased sympathetic and decreased parasympathetic response. • Rapid, shallow breathing  increase venous return • Renin, vasopressin, epinephrine release  increase vasoconstriction, • Glycogenolysis  fluid shift • Decreased organ blood flow (particularly kidneys) • Decompensatory : Reduced organ blood flow  Drive to reduce arterial pressure  positive feedback cycle • These decompensatory mechanisms are compounded by a reduction in sympathetic drive and a change from vasoconstriction to vasodilation with a further lowering of blood pressure. Can lead rapidly to death.
  • 235. Outline the initial management in the treatment of cardiogenic shock. • Early cardiac catheterization and revascularization can improve prognosis prior to occurrence. • Tx of shock: • IV inotropic agents Dobutamine, Dopamine • Increase contractile force • Arterial vasodilators Hydralazine and Ca2+ Channel Blockers • Once BP improves to reduce resistance to LV contraction • Placement of intra-aortic balloon pump into aorta via femoral artery • Percutaneous Left Ventricular Assist Device (LVAD)
  • 236. Main difference of cardiogenic shock vs. noncardiogenic shock physiology is that cardiogenic shock causes decreased CO by DIRECTLY IMPACTING THE CONTRACTILITY OF THE MYOCARDIUM while the other forms of shock indirectly impact CO.
  • 237. Describe how the susceptibility to infarction varies between the myocardial layers. • Transmural infarcts • Span the entire thickness of the myocardium • Result from total, prolonged occlusion of an epicardial coronary artery • Subendocardial infarcts • Exclusively involve innermost layers of the myocardium (usually of the left ventricle, ventricular septum, or papillary muscles) • Subendocardium is MORE susceptible to ischemia because: • It is the zone subjected to highest pressure from the ventricular chamber • It has few collateral connections that supply it • It is perfused by vessels that must pass through layers of contracting myocardium
  • 238. Factors Determining Infarct Size • The amount of tissue that infarcts relates to: • The mass of myocardium perfused by the occluded vessel • The magnitude and duration of impaired coronary blood flow • The oxygen demand of the affected region • The adequacy of collateral vessels that provide blood flow • The degree of tissue response that modifies the ischemic process
  • 239.
  • 240. Arrhythmias • Occur frequently during acute MI and are a major source of mortality prior to hospital arrival • Upon arrival, arrhythmia associated deaths are uncommon • Due to anatomic interruption of blood flow to conduction structures • Accumulation of toxic metabolic products • Membrane leaks causing abnormal cellular ion concentrations • Autonomic stimulation • Administration of arrhythmogenic drugs • Detection: EKG
  • 241. Ventricular arrhythmias within 48 hours solely suggest unstable electrical currents, but after 48 hours it is an indication for ICD implantation
  • 242. Pericarditis • Early in post-MI period: Days 1-3 • Cause: inflammation from necrosis/healing spreads from myocardium to pericardium • Detection: • Sharp pleuritic pain • Fever • Pericardial Friction Rub • Resolved with aspirin • Incidence limited by acute reperfusion strategies post MI • Anticoagulants contraindicated
  • 243. Mechanical Complications • LV Papillary Muscle Rupture: • Detection: Loud holosystolic murmur due to severe acute mitral regurgitation  may be fatal • Partial rupture  moderate regurgitation  sxs of heart failure or pulmonary edema • Posteromedial LV papillary muscle is more susceptible • Ventricular Free Wall Rupture (rare/infrequent): occurs within 2 weeks following MI • Necrotic myocardium  free wall rupture hemorrhage into pericardial space  rapid cardiac tamponade (restricts ventricular filling) • Fatal • Detection: Imaging studies  surgical repair • Ventricular Septal Rupture 3-7 days • Blood shunted from LV to RV  congestive heart failure (JVD) • Detection: Loud holosystolic murmur @ Left Sternal Border (transseptal flow) • Use Doppler echocardiography to distinguish between acute mitral regurgitation (or O2 saturation in chambers) • True Ventricular Aneurysm: Late complication (weeks to months after MI) • Weakened ventricular wall due to phagocytic clearance of necrotic tissue outward bulge • Suspected with persistent ST segment elevations on ECG (weeks later) and/or bulge on CXR • Detection: Confirmed by echocardiography
  • 244. Holosystolic Murmurs • VSD • Mitral Regurgitation • Tricuspid Regurgitation
  • 245. Heart Failure • Due to impaired LV contractility (systolic dysfunction) and myocardial stiffness (diastolic dysfunction) • Detection: • Dyspnea • Pulmonary Rales • Third heart sound (S3) • Rx: ACE inhibitors, Diuretics, Beta blockers, • Beta Agonists • Vasodilators
  • 246. Cardiogenic Shock • When >40% of LV mass has infarcted • Decreased Cardiac Output (CO) • Hypotension: Systolic BP <90 mmHg • Low BP  decreased coronary perfusion  increases ischemic damage  decreased stroke volume increased LV size  enhances myocardial oxygen demand • Mortality >70%
  • 247.
  • 250. ESPVR and EDPVR • The end-diastolic pressure-volume relationship is the passive filling curve of the ventricle, the slope of which is the inverse of ventricular compliance. (A completely stiff, non-compliant ventricle would have a really steep slope.) • The end-systolic pressure-volume relationship is the max pressure that the ventricle can muster for a specific volume given a specific inotropic state. The PV curves cannot “cross” this limit.
  • 251. Preload • Preload is the initial stretching of cardiac myocytes prior to contraction. EDV and EDP are used as estimates. • Increasing venous return and ventricular preload leads to an increase in SV.
  • 252. Determinants • Factors directly proportional to preload • Venous pressure - Increase in venous BP → more atrial filling → more ventricular filling • Ventricular compliance - increased compliance, increased ventricular filling at given pressure • Atrial inotropy - increase in atrial contraction by sympathetic activation can enhance ventricular filling • Outflow resistance -Increase in outflow resistance impairs the ability of the RV to empty  Increase in preload. • Examples: Pulmonic valve stenosis, pulmonary hypertension, aortic valve stenosis, elevated aortic pressure • Factors inversely proportional to preload • Heart rate - Increased heart rate → Less time in diastole → less time for filling → lower preload. • Inflow resistance - Elevated inflow resistance reduces the rate of ventricular filling and decreases ventricular preload • Example: Tricuspid valve stenosis, mitral valve stenosis • Ventricular inotropy - Reduced inotropy → higher end-systolic volume → blood “backs up” in ventricle and proximal venous circulation → increased preload • Venous compliance - i.e. venodilation by NO increases compliance, reducing preload and O2 demand.
  • 253. Ventricular Compliance • Compliance = ΔV/ΔP, but usually we think of it as the inverse of the slope of passive filling on a PV curve. • Ventricular compliance is determined by the physical properties of the tissues making up the ventricular wall and the state of ventricular relaxation.
  • 254.
  • 255. Length Tension Relationship • Length ∝ tension. When a myocyte is stretched, it’s passive tension increases (like a stretched rubber band) and it’s active tension increases (more forceful contraction when electrically stimulated). This means that increasing preload aka End Diastolic Pressure will create a bigger End Systolic Pressure (afterload).
  • 256. Describe the three possible explanations for length-dependent activation. • Increased sarcomere length sensitizes troponin C to calcium. • Fiber stretching alters calcium homeostasis • Actin and myosin are brought in closer proximity to each other
  • 258. Afterload • Afterload- The load against which the heart must contract to eject blood. • Left ventricle afterload ~ aortic pressure • Right ventricle afterload ~ pulmonary artery pressure
  • 259. Wall Stress • La Place Equation: Wall stress, σ, is the average tension each myocyte must produce to shorten against the intraventricular pressure. Wall stress is directly proportional to afterload. • P = intraventricular pressure, r = ventricular radius, h = wall thickness • Intraventricular pressure is directly proportional to wall stress • Ventricular chamber dilatation increases wall stress (increased R) • Hypertrophy decreases wall stress (increased H)
  • 260. Afterload and Velocity of Contraction • Afterload is like the weight that the muscle must lift, so increasing afterload (force) decreases velocity of contraction.
  • 261. Preload and Velocity of Contraction • Increasing preload increases velocity of contraction for a given afterload. However, note that the y-intercepts are equal, meaning that the theoretical max velocity of contraction (what would occur against zero force) remains the same.
  • 262. Afterload and Stroke Volume • At a given preload, increasing afterload decreases stroke volume.
  • 263. What is a normal ejection fraction •55%
  • 264. With changes in preload, EDV will change and with changes in afterload, ESV will change.
  • 265. Inotropy • Increased Inotropy  Increased Stroke Volume
  • 266.
  • 267. Inotropy and ESVPR • Increased inotropy shifts ESPVR (end systolic pressure volume relationship) to the left and makes it steeper because the ventricle can generate increased pressure at any given volume • Increasing Inotropy also increases stroke volume and ejection fraction (EF)
  • 268. Determinants of Inotropy • Sympathetic nerve activation: Sympathetic nerves release norepinephrine which binds to β-1 adrenoceptors on myocytes and play a role in ventricular and atrial inotropic regulation • Circulating catecholamines: have positive inotropic effects. • Afterload: an increase in afterload can cause modest increase in inotropy by a somewhat unknown mechanism. • Heart rate: increased heart rate has positive inotropic effects. • This is due to an inability of the Na+/K+ ATPase to keep up with the sodium influx at higher frequency of action potential at elevated heart rates leading to an accumulation of intracellular calcium via sodium calcium exchanger.
  • 270. Dynamic effects of Increased Preload • Primary change: Increased EDV and SV (right shift, solid red line) • Secondary change: Increased afterload (due to increased CO and BP). • Inotropy is not affected. ESV also increases slightly due to higher afterload.

Editor's Notes

  1. Foam cells, fibrous cap, simple atheroma, complicated atheroma, thrombosis, hemorrhage
  2. Myocardial Infarction, stenosis, ulceration/gangrene, atheroembolism (look for clear spaces, cholesterol clefts, often found in kidney), aneurysm
  3. Very commonly used (1st line for hyperlipidemia) SREBP = sterol regulatory element binding-protein Pravastatin, rosuvastatin, and fluvastatin are not dependent on cyt P450 as much, so less likely to cause myopathy in combination w/ other drugs LO 10, 12, 13
  4. Less commonly used (2nd line for hyperlipidemia) NPC1L1 = Niemann-Pick C1-like 1 LO 10, 12
  5. Rarely used (3rd line for hyperlipidemia) CYP7A encodes cholesterol 7-alpha hydroxylase, which is the rate-limiting step in bile acid synthesis. FXR = farnesoid X receptor; bile acids bind and increase FXR’s transcriptional activity. FXR induces repressors of transcription that inhibit expression of CYP7A (so when there are enough bile acids, FXR represses transcription of the enzyme, CYP7A, that makes bile acids). LO 10, 12, 13
  6. Somewhat commonly used, going out of style (indicated for primary hyperlipidemia, low HDL) Niacin makes it so the liver takes cholesterol from HDL particles but does not actually take up the HDL and destroy it by breaking down its apoprotein, apo AI. LO 14, 15
  7. Commonly used (indicated for hypertriglyceridemia, 2nd line for hypercholesterolemia, used when HDL is low and LDL is high) PPAR = peroxisome proliferator activated receptor alpha. When FAs are high, PPAR induces genes for FA oxidation, which decreases TG production, inhibiting VLDL synthesis. Also enhances expression of lipoprotein lipase (LPL) LO 13, 17
  8. Commonly used (severe hypertriglyceridemia, esp pancreatitis. Used when HDL is low and LDL is high) LO 17
  9. SA node, AV node and bundle of His all supplied by RCA, posterior/inferior infarctions  serious arrhythmias
  10. Prodromes are a good sign. Palpitations or no signs are concerning